AT BOC Prep Domain I - Injury and Illness Prevention and Wellness Promotion

Réussis tes devoirs et examens dès maintenant avec Quizwiz!

B: This test requires participants to run a distance of 1.5 miles (2.4 km) as quickly as possible. The total time to cover this distance is recorded as the outcome measure. Classifi cations of cardiovascular fi tness (based on normative values in particular age groups) have been established and can be used in interpreting outcome data. Predicted Vo 2 max can be calculated from the 1.5-mile run time.

. The Cooper 1.5 mile run test is commonly used to assess aerobic fi tness. What outcome data are produced from this assessment? A. Maximal heart rate B. Minutes run C. Distance (miles or kilometers) run D. Sweat rate

D: Meta-analyses are the most rigorous type of research study and introduce the least amount of bias and risk of error.

16. When evaluating the strength of the evidence found in the research literature, which type of research study would be considered the most rigorous and provide the least amount of bias? A. Case study B. Clinical practice guidelines C. Cohort study D. Meta-analysis

C: The deltoid muscle acts to abduct the shoulder, and its strength can be tested through manual resistance of abduction

19. When performing the quick orthopedic screening as part of the preparticipation examination, the athlete is asked to abduct the shoulders to 90° and hold that position against downward pressure by the examiner. What is being tested during this part of the screening? A. Trapezius strength B. Shoulder range of motion C. Deltoid strength D. Shoulder symmetry

B: This condition is contagious. Fungicide followed by wound coverage minimizes the risk of transmission

A basketball player has recently been diagnosed with tinea corporis and started on a topical fungicide. Under what conditions will the athlete be allowed to return to play? A. The athlete can play as long as the area is covered. B. The athlete can return to play after using the fungicide for 72 hours if the lesion is covered by a gas-permeable membrane and tape. C. The athlete can return to play if symptoms of the virus, such as fever and fatigue, have disappeared and no new lesions have been observed for the past 72 hours. D. The athlete can return to play after being on antibiotics for 24 hours and covering the lesion with a gas-permeable membrane and tape

D: Ozone levels are at their highest during sunny days with high air temperatures. The levels rise during dawn and peak midday. As the day progresses, levels reduce as traffi c decreases along with lowering air temperatures and the setting sun

A city is hosting a 10K race to benefi t at-risk innercity youth. The race course includes running through the city ' s developed and highly populated sections. At what time during the day should the race begin to best minimize the risk of runners experiencing problems due to air pollution? A. Late dawn B. Midday C. Early Afternoon D. Evening

D: The National Athletic Trainers' Association (NATA) position statement on lightning safety for athletics and recreation recommends activity be postponed or suspended if a thunderstorm appears imminent

A comprehensive lightning emergency action plan includes identifying specifi c criteria for suspending and resuming activity. Which of the following is a recommendation for suspending or resuming activity? A. Resume activities once skies brighten and the sound of thunder lessens. B. Postpone or suspend activities once thunderstorms are producing lightning. C. Resume activities 30 minutes after all persons are completely within the designated safe location. D. Postpone or suspend activities if a thunderstorm appears imminent before or during activity

C: Off-the-shelf orthotics are easily implemented and offer a chance to determine if addressing the biomechanical issues will provide therapeutic benefi ts.

A cross country runner has been complaining of symptoms consistent with tibialis posterior tendonitis. Evaluation reveals some very mild increased pronation with ambulation. The team physician recommends symptomatic treatment and an off-the-shelf arch support before fabrication of custom orthotics. What is the basis for this recommendation? A. Custom orthotics are not very effective with minor biomechanical abnormalities. B. Custom orthotics do not provide enough cushioning for a cross country runner. C. Off-the-shelf orthotics are a more-cost effective way to determine if custom orthotics would be helpful. D. Off-the shelf orthotics are easier to replace, as this athlete will be participating in daily rigorous workouts.

C: If symptoms have resolved and liver function and spleen size are normal, a return-to-activity progression can begin

A female cross country runner has been recently diagnosed with mononucleosis. How long must the athlete be withheld from activity before being reevaluated to determine if it is safe to begin a return to activity progression? A. 7 days B. 14 days C. 21 days D. 28 days

D: As stated in recommendation 28 of the National Athletic Trainers' Association (NATA) position statement, the athletic trainer ' s role is to be the athlete ' s advocate

A female lacrosse player is exhibiting signs of disordered eating. According to the National Athletic Trainers' Association (NATA) position statement on preventing, detecting, and managing disordered eating in athletes, the certifi ed athletic trainer plays an important role in managing an athlete who is being treated for an eating disorder. Which intervention is within the recommended role of the certifi ed athletic trainer? A. Determine limitations of physical workouts based on team practice schedules. B. Make the initial diagnosis and design a treatment plan. C. Be willing to work individually with athletes with disordered eating who do not want to seek treatment from physicians or psychotherapists. D. Assume the role of informed patient advocate

B: Lyme disease is a tick-borne disease characterized by erythema chronicum migrans—an enlarging red ring that may or may not have a center raised papule

A few days after returning from a trail run one of your cross country runners complains of headache, fatigue, and fever followed by a red ringlike rash with a red center. What condition do you suspect? A. Ringworm B. Lyme disease C. Spider bite D. Rocky Mountain spotted fever

B: The seal indicates to consumers that the helmet met industry standards at the time of manufacture or reconditioning

A football helmet bears the National Operating Committee on Standards for Athletic Equipment (NOCSAE) seal. What does this seal imply about the helmet? A. It is a warranty that the helmet will protect the athlete from potential head injuries. B. This helmet met the requirements of NOCSAE performance testing when it was manufactured or reconditioned. C. The helmet will minimize the severity of a head injury such as a concussion. D. It states that there is risk inherent in playing football and that a serious injury can occur as a result of participation in the sport. E. The helmet can be safely removed during an on-fi eld emergency care situation.

D: Consensus statements and position statements are available to provide evidence-based recommendations for clinical practice and are based on the work of experts in the fi eld.

A hospital has hired you to provide outreach coverage by directing the healthcare for a local recreation department that organizes youth sports leagues for 10 different sports. What resource might provide you with recommendations for implementation of care based on evidence-based principles? A. Meeting with the hospital ' s legal team B. Gathering together pediatricians in the area to discuss required care C. National Recreation and Parks Administration D. Inter-Association Consensus Statement on Best- Practice Recommendations for Youth Sports Leagues

D: Exercise-induced bronchospasm results from hyperreactivity of the athlete ' s airway. Moist and temperate air passing through the airway may elicit less of an airway reaction compared with other environments.

A middle-distance runner has exercise-induced asthma. Which of the following environments is most likely to limit the intensity and number of wheezing episodes during activity? A. Cold and dry B. Temperate and dry C. Cold and moist D. Temperate and moist E. Hot and dry

C: A medical history form should be completed before the physical examination in order to identify any past medical problems and any need for referral to medical professionals, counselors, and other health-care professionals

A negative ("no") response to which of the following questions on a preparticipation examination medical history form would warrant referral to a health-care professional for additional screening? A. Do you limit or carefully control what you eat? B. Has anyone recommended you change your weight or eating habits? C. Are you happy with your weight? D. Are you trying to gain or lose weight? E. Are you taking any medications or supplements for weight loss or weight gain?

B: A peak fl ow meter assesses forced expiratory ability by measuring how air comes out of the lungs when exhaling forcefully after inhaling fully

A peak fl ow meter is an asthma screening tool that can be used to help quickly assess asthma severity and determine the effectiveness of medications. What instructions should be provided to the patient using a peak fl ow meter? A. Slide the indicator to the base of the meter, place the peak fl ow meter mouthpiece in your mouth, and after sealing your lips tightly around the mouthpiece inhale completely until the indicator rises within the meter. B. After placing the peak fl ow meter mouthpiece in your mouth, seal your lips tightly around the mouthpiece, then blow out as hard and as fast as you can one time. C. Stand or sit up straight, exhale completely, then with your lips tightly around the mouthpiece inhale as fast as you can one time. D. While standing, slide the indicator to the base of the meter and then wrap your lips tightly around the mouthpiece, take in a deep breath, and maximally exhale as slowly as possible.

A: Based on recommendations from the American Heart Association, a history of symptoms during exertion warrants referral to a physician.

A positive response to which of the following questions on a preparticipation examination medical history would result in referral to a physician for additional screening? A. Does your heart race or skip beats during exercise? B. Have you ever spent the night in a hospital? C. Is there anyone in your family who has asthma? D. Have you ever fractured a bone?

A: Strength Of Evidence Taxonomy (SORT) and Centre for Evidence-Based Medicine (CEBM) scales provide means to evaluate the level of evidence and confi dence of the evidence-based recommendation being made

A recently released position statement recommendation carries a Centre for Evidence-Based Medicine (CEBM) and a Strength Of Evidence Taxonomy (SORT) rating of "A." How would a clinician best apply this recommendation? A. The clinician should attempt to apply this recommendation into clinical practice, as it carries a high degree of evidence. B. The clinician should evaluate how the recommendation might fi t in with current clinical practice, as it carries a moderate level of evidence. C. The clinician should carefully consider how this recommendation might fi t in with current clinical practice, as it carries weak or confl icting evidence. D. The clinician would not want to immediately apply this to clinical practice, as it carries insuffi cient evidence to make a recommendation.

E: A dipstick urinalysis is considered a reliable method for determining the presence of a urinary tract infection, as the presence of nitrates or leukocyte esterase confi rms the diagnosis.

A routine dipstick urinalysis is often part of a preparticipation physical examination. The following are the results of a urinalysis: - Appearance = light yellow and cloudy - Leukocyte esterase (LE) = moderate - Nitrite = positive - Blood = nonhemolyzed moderate What condition do these urinalysis results suggest? A. Dehydration B. Diabetes mellitus C. Disordered eating D. Blocked bile duct E. Urinary tract infection

C: Total fi ber is the combination of functional fi ber, nondigestible carbohydrate that may be added to foods, and dietary fi ber, classifi ed as either soluble or insoluble fi ber

A runner who is trying to improve the quality of her carbohydrate intake asks you to explain the fi ber information on the two energy bars she is comparing so she can make an educated choice. Which statement is correct? A. Soluble (high-viscosity) fi ber is a digestible carbohydrate found naturally in foods that increases fecal bulk and acts as a laxative speeding up the movement of food through the digestive system. B. Insoluble (low-viscosity) fi ber is a digestible carbohydrate found naturally in foods that slows gastric emptying, increases the feeling of fullness, and may aid in decreasing blood cholesterol levels. C. Functional fi ber is nondigestible carbohydrate that may have benefi cial physiological effects and is typically added to foods. D. A food label stating it is "high fi ber" food means one serving of the product contains the recommended daily allowance of fi ber.

B: The literature supports wearing a shoe that does not "stick" to the synthetic turf to signifi cantly reduce the likelihood of injury

A school that has recently built a new football practice and game facility has decided to have their lacrosse team use the old synthetic turf football fi eld. Which of the following policies can aid in minimizing the lacrosse players' injury risk? A. All training sessions should occur on the synthetic turf fi eld. B. Players should wear shoes that provide solid footing without "sticking" to the surface. C. Players should wear shoes with a soft, fl exible sole. D. Practice sessions should be conducted at 50% to 75% of maximal intensity and speed.

B: The Generalized Anxiety Disorder-7 (GAD-7) is a standardized measure that can be used as a brief assessment tool for assessing generalized anxiety disorder

A screening tool asks the patient to indicate how frequently over the past 2 weeks he has been bothered by the following: (1) feeling nervous, anxious, or on edge; (2) not being able to stop or control worry; (3) worrying too much about different things; (4) trouble relaxing; (5) being so restless that it is hard to sit still; (6) becoming easily annoyed or irritable; (7) feeling afraid as if something awful might happen. Which mental health condition is this tool assessing? A. Attention defi cit-hyperactivity disorder B. Generalized anxiety disorder C. Obsessive-compulsive disorder D. Depression

D: PECC, the Protective Eyewear Certifi cation Council, works to reduce eye injuries through certifi cation of protective eyewear.

A seal from this organization appears on eye protection, such as eye guards, manufactured in compliance with American Society for Testing and Materials (ASTM) International Standards. A. NOCSAE B. HECC C. CSA D. PECC

B: Athletes who are forced to sit out for any reason during the initial heat-acclimatization period resume the protocol where they left off even if it is different from the rest of the team.

A secondary school athlete is forced to sit out for days 5 and 6 of the preseason initial 14-day heat acclimatization period due to injury. What day of the protocol will this athlete be on when he returns to practice? A. Day 7 B. Day 5 C. Day 4 D. Day 1

A: Pediatric athletes should increase the intensity and duration of their workouts by no more than 10% each week to allow adaptation and reduce risk of overload

A seventh-grade runner participating on a cross country team is running approximately 10 miles per week at the beginning of team practices. How many miles a week can the coach safely have the athlete run in the second week of practice? A. 11 miles B. 12 miles C. 15 miles D. 20 miles

D: Athletes participating in sports with risk for orofacial injury should be encouraged to wear properly fi tted mouth guards.

A soccer coach is seeking support for his decision to require all his players to wear mouth guards during both practice sessions and matches. Based on the National Athletic Trainers' Association (NATA) position statement on preventing and managing sports-related dental and oral injuries, which statement regarding mouth guard use in sports is accurate? A. Mouth guards are effective in preventing mild traumatic brain injury. B. Mouth guards negatively affect ventilation and aerobic performance. C. Mouth guards have been shown to improve strength and balance. D. Mouth guards are effective in protecting against orofacial injuries.

C: Increased sensitivity to ultraviolet (UV) rays is a side effect of the antibiotic tetracycline. Sunblock is ineffective in preventing this reaction.

A soccer player reports that his physician has recently started him on a prescription for tetracycline to manage acne. Which of the following is an important consideration based on the new medication? A. The athlete is now at a higher risk for heat illness. B. The athlete should increase fl uid intake to combat the diuretic effect of the medication. C. The athlete is at risk for photosensitivity to sun exposure. D. The athlete should have liver function tests completed every 3 months.

A: While the research does not conclusively indicate synthetic surfaces are more likely to cause injuries compared with natural surfaces, both abrasions and turf toe (hyperextension of fi rst metatarsophalangeal joint) seem to occur more frequently in athletes using synthetic surfaces.

A soccer team will be playing on synthetic turf during an upcoming 2-day tournament. What injuries should you be most prepared to treat during this tournament? A. Abrasions and turf toe B. Concussions and arch sprains C. Contusions and ankle sprains D. Tibial fractures and concussions E. Blisters and hamstring strains

A: When new to a high-altitude environment, athletes with known sickle cell trait (SCT) should have access to supplemental oxygen for training sessions and competitions.

A sports team travels to a city located at a higher altitude for an end-of-season tournament. An athlete with which of the following conditions should be closely monitored for effects resulting from the change in altitude? A. Sickle cell trait B. Urticaria C. Celiac disease D. Mittelschmerz

E: This response is consistent with the description of the condition

A swimmer at a high school for which you are the athletic trainer has been diagnosed with Wolff- Parkinson-White syndrome. Which statement best characterizes this condition? A. Currently the leading cause of sudden death in athletes B. Characterized by a prolonged P-R interval and a shortened QRS complex C. Results in atrial pre-excitation and bradycardia due to accessory pathway electrical activity D. Results in delayed atrial excitation and tachycardia due to an electrical accessory pathway E. Characterized by ventricular pre-excitation and tachycardia due to electrical conduction over accessory pathways

B: This is in compliance with the rules for mandatory equipment as outlined by the National Collegiate Athletic Association (NCAA) Rules Committee

According to National Collegiate Athletic Association (NCAA) guidelines, what organization must certify shin guards worn by soccer players? A. American Society for Testing and Materials (ASTM) B. NCAA Committee on Competitive Safeguards C. National Operating Committee on Standards for Athletic Equipment (NOCSAE) D. Soccer Equipment Certifi cation Center (SECC)

A: The National Collegiate Athletic Association (NCAA) guidelines state that an athlete at any division level should complete a preparticipation medical evaluation on entrance to the institution

According to National Collegiate Athletic Association (NCAA) guidelines, which of the following collegiate athletes must complete a comprehensive preparticipation medical evaluation? A. A baseball player transferring from a similardivision NCAA institution who reports no injuries or health concerns on his medical history form B. A junior basketball player who has not completed a comprehensive examination since his freshman year C. A junior football player who has not completed a cardiovascular screening as part of the preparticipation examination since his freshman year D. A senior sprinter who had only history and blood pressure measurements taken as part of the cardiovascular screening each season since the initial preparticipation physical examination E. A senior fi eld hockey player who tore her anterior cruciate ligament in her sophomore season and participated during her junior year

B: The pathology combined with the level of contact inherent in the sport is consistent with the recommendation from the American Academy of Pediatrics

According to the American Academy of Pediatrics Guidelines for Sports Participation, which of the following athletes should be recommended for clearance to participate? A. An athlete recently diagnosed with carditis wishing to participate in lacrosse B. An athlete with atlantoaxial instability wishing to participate in cross country C. An offensive lineman with one kidney D. An athlete with an enlarged spleen wishing to participate in swimming E. An athlete with an enlarged liver wishing to participate in basketball

B: According to the National Athletic Trainers' Association (NATA) position statement, the internal layer should allow for sweat evaporation with minimal sweat absorption, the middle layer should provide insulation, and the external layer should be wind and water resistant, while allowing for evaporation of moisture

According to the National Athletic Trainers' Association (NATA) position statement on environmental cold injuries, three layers of clothing are recommended to minimize risk of hypothermia. Which of the following is correct regarding these layers? A. The external layer allows evaporation of sweat with minimal absorption. B. The internal layer allows evaporation of sweat with minimal absorption. C. The middle layer provides resistance against wind. D. The external layer provides insulation. E. The internal layer provides insulation.

A: The Occupational Safety and Health Administration (OSHA) established standards that govern employee occupational exposure to bloodborne pathogens. These standards and guidelines were developed to protect all employees in health-care settings, health-care providers, and patients

According to the Occupational Safety and Health Administration (OSHA) Bloodborne Pathogen Standard, which statement best describes the persons required to receive training through their employer in regard to occupational exposure to bloodborne pathogens? A. Any employee who has the potential for occupational exposure to blood or other potentially infectious materials B. Only employees involved in direct patient care who might be exposed to blood or other potentially infectious materials C. All federal and state employees D. Only health-care providers dealing with high-risk patient populations E. Persons completing a fi rst-aid and cardiopulmonary resuscitation course

C: Intense exercise in a hot and humid environment can cause acute exertional rhabdomyolysis in a healthy person. Symptoms include gradual onset of muscle weakness, swelling, pain, renal dysfunction, and presence of dark urine

Acute exertional rhabdomyolysis can occur in healthy individuals during intense exercise. What environmental conditions increase an athlete ' s risk for this condition? A. High air temperature and rain B. Low air temperature and wind C. High air temperature and high humidity D. Low air temperature and high humidity E. Low air temperature and rain

C: Transference refers to moving all or a portion of well-defi ned risks of either high fi nancial risk but low frequency or high frequency but low fi nancial risk to an outside entity

After evaluating data for the past 5 years on injury rates and medical expenses incurred by the athletic department, the institution elects to purchase a secondary insurance policy to cover those expenses. What type of risk management strategy is being utilized? A. Avoidance B. Retention C. Transference D. Reduction

A: Severe muscle cramping, heat intolerance, level breathing, tachycardia, and hypotension can be associated with exertional sickling. Sickle cell trait is more common in certain ethnic groups, including African Americans

An African American athlete reports for her preparticipation examination and notes she has experienced heat intolerance, severe muscle cramping, hyperventilation, tachycardia, and symptoms of hypotension during the past 6 months. Based on this information, what action should you take? A. Screen for sickle cell trait B. Screen for hypertension C. Screen for Marfan ' s syndrome D. Screen for type 2 diabetes mellitus E. Assess her overall fi tness level

A: Purity and safety of ingredients in dietary supplements are not regulated by any agency and should never be assumed to be safe.

An athlete approaches you for guidance regarding a dietary supplement he wishes to take to improve performance. What information would it be important for you to share with the athlete? A. Dietary supplements are not regulated by the U.S. Food and Drug Administration (FDA), so the product may contain substances that could result in a positive drug test. B. The ingredient list should be carefully evaluated for banned substances. C. The team physician is required to approve all supplements before use. D. The student should consult with his coach who has a list of all products that are approved for use by the National Collegiate Athletic Association (NCAA).

B: Turf toe or a hyperextension sprain of the great toe commonly occurs due to the fl exibility of turf shoes that athletes wear when practicing or competing on synthetic turf

An athlete complains of pain and swelling that is isolated to the metatarsophalangeal joint of the great toe. Pain is greatest when pushing off during walking and running and began after the athlete started wearing new turf shoes to practice on the synthetic turf fi eld. What injury would you suspect? A. Sesamoiditis B. Turf toe C. Morton ' s neurom D. Jones Fracture

D: Passive motion is used to assess noncontractile structures and flexibility of contractile tissues being put on stretch. When the ankle joint is passively dorsiflexed, the ankle joint plantar flexors are put on stretch.

An athlete demonstrates limited passive dorsiflexion at the preparticipation screening. Which of the following factors is most likely restricting this motion? A. Strength of the posterior tibialis muscle B. Strength of the peroneal muscles C. Flexibility of the toe extensor muscles D. Flexibility of the Achilles tendon complex E. Strength of the anterior tibialis muscle

B: Reporting certain infectious diseases is aimed at (1) isolating the disease and retarding its spread and (2) generating data to be used for future disease prevention programs. For most infectious diseases the disease, not the person, is reported. However, with a sexually transmitted infection (STI) the infected person and all partners are identifi ed so that they can be contacted

An athlete diagnosed with a sexually transmitted infection (STI) tells you she is upset because the physician told her that her infection must be reported to the public health authorities. What should be communicated to this athlete? A. Certain infectious diseases are reported so that pharmaceutical companies can target distribution efforts to geographical areas. B. Some infectious conditions are reported to protect the public from an outbreak of the disease. C. Certain infectious diseases are reported to encourage patients to improve personal hygiene and incorporate safe-sex practices. D. Some infectious conditions are reported so that hospitals can make decisions for allocating funding for health-care personnel.

A: An individual with a substance abuse or mental health condition is ready to assist in developing a treatment plan once he or she moves from the precontemplation stage to the contemplation and preparation stages

An athlete is demonstrating a pattern of illegal substance abuse and dependence. While having a discussion with this athlete about his options, you employ Prochaska and DiClemente ' s framework to determine his readiness to do something about his condition. Which of the fi ve different stages of change indicates he is not ready to work with you to develop a treatment plan? A. Precontemplation B. Contemplation C. Preparation D. Action

C: Mumps is highly contagious for persons who have not been vaccinated. The infected person should be isolated and treated symptomatically, and others should be made aware of the symptoms of the disease.

An athlete on your women ' s basketball team is diagnosed with the mumps. How would you best manage this condition to reduce the risk for other members of her team? A. Send the athlete home for a minimum of 25 days and educate the other members of the team about the symptoms of the disease. B. Provide antiviral medication for everyone who has been exposed and isolate the sick athlete so that no additional exposure will occur. C. Isolate the sick athlete until parotid swelling returns to normal, verify immunization records and vaccinate other team members if necessary, and educate anyone who has been exposed about the symptoms. D. Treat the athlete symptomatically with analgesics and rest, verify the immunization record of anyone else who has been exposed, and provide vaccinations for those who have not been vaccinated.

C: Five disciplines have been identifi ed by the U.S. federal government to be competent to provide mental health services. Psychiatrists and psychiatric nurses both take a medical approach to treatment, including the use of medications

An athlete presents with a potential major mental health disorder, and medical intervention is needed. Referral to which of the following professionals is most appropriate? A. Social worker holding an MSW degree B. Psychologist holding a PsyD degree C. Psychiatrist holding a DO degree D. Psychologist holding a PhD degree

E: The purpose of the brace is to limit tensile forces placed on the medial collateral ligament (MCL) and lateral collateral ligament (LCL). Valgus and varus forces are minimized by the medial and lateral supports, respectively

An athlete reports to the athletic training room wearing a neoprene knee brace with medial and lateral supports. What is the intended purpose of this brace? A. To provide support for a patellofemoral condition and enhance proprioception B. To allow for controlled progressive immobilization C. To provide enhanced proprioception to minimize risk of initial injury D. To provide restriction against rotational forces E. To provide additional support subsequent to a collateral ligament injury

D: Positive responses to mental health-related screening questions should be reviewed and explored by the team physician to determine the need for additional referral.

An athlete responds affirmatively to the following questions on the pre-participation medical history form: (1) I have trouble sleeping. (2) I lack energy many days of the week. (3) I rehash things many times. (4) I am often anxious or very nervous. (5) I lack confidence. (6) I have difficulty managing my emotions. What actions should you take based on these responses? A. Encourage the student to begin meditation techniques to decrease stress. B. Enroll the student in a stress management class at the counseling center. C. Restrict the student athlete from practices until a full mental health evaluation is completed. D. Alert the team physician so follow-up questions can be asked to gain additional information

D: High-altitude pulmonary edema (HAPE) typically occurs at altitudes from 9,000 to 10,000 ft when fl uid accumulates in the alveolar walls. The condition rapidly resolves with lower altitude and sometimes oxygen implementation

An athlete returns from a family vacation in Breckenridge, Colorado, and reports that she was sick while on vacation. She noted shortness of breath, cough, headache, and general body weakness that was signifi cantly better as soon as they returned home. What condition do you suspect the athlete was experiencing while on vacation? A. Viral infection B. Acute bronchitis C. Acute mountain sickness D. High-altitude pulmonary edema (HAPE)

C: Social workers are trained to connect people with support services and community resources

An athlete shares that his father recently lost his job and his family is having trouble affording food for more than two meals per day. Which community professional would be able to provide the best resources to assist this family? A. Psychologist B. Occupational therapist C. Social worker D. Registered dietitian

D: Uniforms need to be changed only if saturated with blood

An athlete sustains a laceration during a game that results in blood saturating the uniform. How would you properly manage this situation in order to return the athlete to the game? A. The athlete may be returned to the game immediately as long as the wound is no longer actively bleeding. B. The athlete must be removed from competition for the remainder of the game. C. The uniform must be cleaned with an approved disinfectant for fabrics before returning to the game. D. The athlete must change the portion of the uniform that is saturated before returning to the game.

B: Nutritional habits should always be evaluated first, as changes to dietary intake may meet intended outcomes without the use of supplementation. Supplementation combined with improper nutrition may not yield intended results.

An athlete wants to gain lean muscle mass in order to improve his overall performance levels and is considering creatine supplementation. What action should you take first? A. Evaluate the creatine supplement to make sure it does not include any banned substances. B. Assess the athlete's current nutritional habits to see if those need to be adjusted. C. Educate the athlete on the advantages and disadvantages of supplements before approving its use. D. Conduct a thorough evidence-based review of the supplement to assess its efficacy.

D: Athletic trainers should be conscious of their professional limits and scope of practice. When psychosocial symptoms are not being successfully managed the athlete should be referred to a qualifi ed professional

An athlete was recently involved in a motor vehicle accident in which her friend was paralyzed. Since that time the athlete has been very short-tempered and has had trouble sleeping more than 2 to 3 hours per night. The athlete shares that she does not think she can travel in the coming week, as she is terrifi ed to get on the bus. After discussing this situation with the team physician, what would be the best way to manage this situation? A. Refer the athlete to the team massage therapist to help her relax B. Teach the athlete deep breathing and meditation exercises C. Take the athlete through an imagery exercise to visualize a safe bus trip D. Refer the athlete to the school counseling center

C: In the United States, herbal supplements are not closely regulated and have not been as thoroughly studied and evaluated as medications. Use of St. John ' s wort has demonstrated mixed results in treating depression. Additionally, it has been shown to interact with many medicati

An athlete who has been undergoing treatment for depression and anxiety inquires about taking St. John ' s wort, a commonly used herbal supplement, as a more "natural" way to address his mental health conditions. How should you respond to this athlete? A. This supplement is safe to consume because like all medications it is closely regulated. B. Like most herbal supplements, this supplement has been thoroughly studied, and its effects on health are well understood. C. This supplement has been shown to interact with many medications, which could lead to serious side effects. D. If this supplement is purchased in a retail store, it will not be adulterated or contaminated.

B: Repeat testing is recommended at 6 weeks, 3 months, and 1 year because antibodies are not always detectable for some time

An athlete you are working with shares that she was recently a victim of sexual assault. She is working with a counselor to manage the aftereffects of the trauma, but she is very worried that she is at risk for human immunodefi ciency virus (HIV). Initial HIV antibody testing was negative. How would you counsel the athlete based on these results? A. She does not need to worry because she had a negative result from her fi rst test. B. It may take 3 months to a year for measurable antibodies to appear, so she needs to be retested at 3 months and 1 year after the assault. C. If her assailant tests positive for HIV, she will be notifi ed. D. Testing should be repeated monthly for up to 2 years from initial exposure.

A: Whenever hands or any skin surfaces come in contact with blood or bodily fl uids, they should immediately be washed with soap and water or antigermicidal agents.

An athletic trainer is exposed to a patient ' s blood before applying gloves. After providing medical assistance to the patient, what should be this athletic trainer ' s fi rst step in minimizing her occupational exposure to a bloodborne pathogen? A. Cleanse the exposed body area with soap and water or other antigermicidal agents. B. Inform a supervisor about the exposure incident. C. Seek medical care within 2 hours as designated in the exposure control plan. D. Request medical information or testing from the treated patient. E. Complete an exposure report.

A: Due to increased research on concussion management, more than 40 states have youth sports safety laws related to concussion management.

An athletic trainer working at the secondary school level may elect to send concussion awareness information and return-to-play requirements home with each athlete before the start of each sports season. What legislative actions over the past decade have prompted this educational effort? A. The majority of states have enacted sports safety laws related to removal from participation, evaluation, and return-to-play requirements after concussion. B. Parent organizations and booster clubs have lobbied for educational efforts. C. State athletic training practice acts require patient education following concussion. D. National high school athletics governing bodies require patient and family education related to concussions.

A: A fl ared heel is essential to absorb the heelstrike shock, and a curved last is for persons with abnormally high arch and who supinate (run on outside of foot) because a curved last has a larger curve on the medial side of the shoe and a wider outside portion. Cross country runners typically run a high number of miles each week, with an average of 1,500 heel strikes per mile. Thus, a shoe with a moderate-to-high degree of shock absorption is essential

An evaluation of a cross country runner reveals that she has excessive supination bilaterally. Her coach would like to purchase shoes that most effectively address her biomechanical alignment. Which type of shoe would you recommend for this athlete? A. Shoe with a fl ared heel, curved last, and moderate-to-high degree of shock absorption B. Shoe with little to no fl are, straight last, and moderate-to-high degree of shock absorption C. Shoe with a fl ared heel, straight last, and low-tomoderate degree of shock absorption D. Shoe with no heel fl are, curved last, and moderate-to-high degree of shock absorption E. Shoe with a fl ared heel, board last, and low-tomoderate degree of shock absorption

D: The football players are most at risk due to the amount of protective equipment they wear and the strenuous nature of their physical activity

As a high school athletic trainer, you have multiple practice sessions occurring simultaneously. Which group of athletes is most at risk for developing a heat-related illness? A. Cross country team completing speed drills B. Field hockey team preparing for tomorrow ' s game C. Soccer team working on corner kicks D. Football team running through offensive plays E. Volleyball team completing conditioning drills

C: Risk of injury is determined by the type of sport, with collision/contact sports, such as lacrosse, presenting the most risk; limited contact sports, such as softball, presenting less risk of injury; and noncontact sports, such as tennis, presenting the lowest risk of injury.

As a result of rained out games earlier in the week, your high school is scheduled to host a boy ' s lacrosse match, a softball game, and a boy ' s tennis match simultaneously this afternoon. As the sole athletic trainer at your high school, what should you communicate to the coaches of these three teams? A. Based on the inherent risk of injury in these sports, you will be splitting your time equally between all three events. B. Based on the inherent risk of injury in these sports, you will be splitting time between the lacrosse match and the softball game. C. Based on the inherent risk of injury in these sports, you will be at the lacrosse match during the entire match. D. Based on the inherent risk of injury in these sports, you will be at the softball game for the entire game.

A: Administrative control measures include changes to work schedules and establishment of work policies

As an athletic trainer working for a large manufacturing company, you have been tasked with conducting an ergonomic risk assessment (ERA) for the workers who move the boxed product from the assembly line to the shelves in the storage room. From your ERA, you have identifi ed risk factors to be controlled to minimize risk of workplace-related injury. You have recommended a maximum box size for moving by a single person. What type of control measure have you recommended? A. Administrative changes B. Physical changes C. Personal protective equipment changes D. Wellness changes

A: Tanner ' s fi ve stages of maturity begin with the fi rst stage, whereby there is no evidence of puberty to the fi fth stage where pubic hair and genitalia development indicates full maturity

As part of the general medical screening, a maturity assessment is conducted using Tanner ' s fi ve stages of maturity. A female athlete is assigned a stage 5 rating. How should this staging be interpreted? A. This athlete is fully developed. B. This athlete is midway to full development, and growth plates are weaker than joint capsule and tendons. C. This athlete is in the stage when the fastest bone growth occurs. D. This athlete demonstrates no evidence of puberty

A: The reference range for urine specifi c gravity is 1.005-1.030. Specifi c gravity results above 1.010 can indicate mild dehydration. The higher the number, the more dehydrated the athlete may be

Assessing urine specifi c gravity may aid in identifying dehydration, which can increase risk for exertional heat illness. Which urine specifi c gravity value is above the reference range for urine specifi c gravity and may indicate dehydration? A. 1.040 B. 1.010 C. 1.005 D. 1.000

A: Acute onset of urticaria may indicate anaphylactic shock. The signs and symptoms in this question also suggest anaphylactic shock. Other signs and symptoms include facial edema, stridor, diffi culty talking, diffi culty swallowing, hoarseness, hypotension, and syncope

At the start of soccer practice as the players are stretching on the fi eld, one player reports that her throat and chest skin suddenly feel very itchy. She also reports her eyes and mouth feeling itchy, her eyes watering, and her tongue feeling thick and swollen. As she is talking you note a change in her breathing indicating respiratory distress. What condition should be in your initial differential diagnosis? A. Anaphylactic shock B. Acute infl uenza C. Acute bronchitis D. Acute sinusitis

A: Laws and policies surrounding the use of therapeutic modalities by athletic trainers are established and regulated at the state governmental level

At what governmental level are the laws and policies regarding the use of therapeutic modalities by athletic trainers established and regulated? A. State B. National C. Regional D. Local E. International

B: Older adults can increase Vo 2 max with aerobic/ oxidative training programs. The intensity of this type of program should be between 60% and 80% maximum heart rate but should be reached through slow systematic progression

At what intensity should older adults participate in a cardiovascular training program aimed at increasing Vo 2 max? A. 70%-90% maximum heart rate B. 60%-80% maximum heart rate C. 35%-55% maximum heart rate D. 20%-40% maximum heart rate

A: The ABCDE ' s of melanoma are asymmetry (A), border (B), color (C), diameter (D), and evolving (E).

Athletes, regardless of skin type, should be educated on the signs and symptoms of the various types of skin cancer and encouraged to conduct regular skin inspections and to immediately report any suspicious lesions. The ABCDE mnemonic can be used as a guide to recognize suspicious pigmented lesions. For this mnemonic, which letter descriptor is correct? A. A stands for asymmetry: a malignant lesion does not have equal top and bottom or left and right sections B. B stands for blackness: a malignant lesion has a uniform black color C. C stands for circumference: a malignant lesion has a circumference greater than 6 mm D. D stands for distinct: a malignant lesion has a distinct border E. E stands for elliptical: a malignant lesion has an elliptical shape

B: Research suggests that standard chest protectors are ineffective in preventing commotio cordis and that safety baseballs may be more effective in reducing ventricular fi brillation resulting from a ball impacting the chest.

Based on laboratory studies, what safety measure appears to be the most effective method for reducing the risk of commotio cordis in youth sports? A. Use of standard lacrosse chest protectors B. Use of safety baseballs C. Use of coach for pitching in baseball and softball D. Use of age-, height-, and weight-appropriate bats in baseball and softball

E: A comprehensive medical and family history should be obtained, which can guide the physical examination. A limited general physical examination is recommended, and the use of routine laboratory and other screening tests is not supported by the literature.

Based on recommendations in the National Athletic Trainers' Association (NATA) Position Statement: Preparticipation Physical Examinations and Disqualifying Conditions, the inclusion of which routine laboratory or screening tests in the preparticipation examination for all participants is supported by current research studies? A. Electrocardiography B. Urinalysis C. Complete blood count D. Lipid profi le E. Heart auscultation

C: One of the primary responsibilities of the athletic trainer is to prevent injuries through maximizing a safe playing environment. Administrators should be notifi ed any time a possibly unsafe environment is identifi ed

Before basketball practice, you notice that the padding surrounding the backboard is beginning to come off. What action should be taken? A. No action needs to be taken until the environment becomes unsafe for participants. B. The loose padding should be removed so it does not come off during practice. C. The administration should be notifi ed so the backboard can be repaired before it becomes unsafe. D. The coach should be notifi ed that the backboard cannot be used for practice until it is repaired

C: PAR-Q stands for Physical Activity Readiness Questionnaire. This questionnaire and the modifi ed American Heart Association/American College of Sports Medicine Health/Fitness Facility Preparticipation Screening Questionnaire are two recommended self-guided preparticipation screening tools.

Before initiating a conditioning program, participants should complete a preparticipation multistage health screening that includes the use of self-guided questionnaires. What health questionnaire should be used for participants aged 15 to 69 years to determine if they should consult a physician before beginning a conditioning program? A. ICD-9 B. PIQ-6 C. PAR-Q D. PHQ-9

C: The cycling helmet should be fi t so that the helmet is level with the head, the "Y" of the front and rear straps sitting just below the ears, and the chin strap comfortably snug to the chin. Once fi t, shaking the head should not create any helmet movement. Additionally, pushing up on the underside of the helmet should not create more movement than an inch or so from level

Cycling helmets are designed to protect the head against a single traumatic impact. Which of the following is a recommended guideline for selecting and fi tting a cycling helmet? A. Place the helmet on the head so that the top of the helmet creates a 15° to 20° angle of incline from front to back. B. Adjust front straps and rear straps to position the "y" created by the straps just above the ears, evenly on each side. C. To check a fi tted helmet, push up under the front edge trying to move the helmet backward and upward, and if it moves more than 1 in. from level, readjust straps. D. Adjust the chin strap so that it sits 2 fi nger widths below the inferior chin.

D: Replacing lost fl uid with appropriately formulated sports drinks has been shown to be more effective than using only water.

Dehydration in an exercising person can increase risk for exertional heatstroke. Athletes who are dehydrated need to replace fl uids. Based on current research, how should fl uid lost during exercise be replaced? A. The athlete should replace fl uid lost by drinking primarily water and avoiding sports drinks. B. The athlete should replace fl uid lost by drinking water until athlete is no longer feeling thirsty. C. The athlete should replace fl uid by drinking more fl uid than fl uid lost. D. The athlete should replace fl uid lost by drinking appropriately formulated sports drinks.

D: Capillaries are single-layered blood vessels that connect arterioles to venules. They are the site for exchange of nutrients and metabolic wastes between the tissues and blood

Distribution of oxygen and usable metabolic materials occurs through which vessels? A. Arteries B. Veins C. Glands D. Capillaries E. Venules

B: The National High School Sports-Related Injury Surveillance Study is an Internet-based data collection tool similar to the National Collegiate Athletic Association (NCAA) Injury Surveillance System. This reporting system collects data from a national sample of high schools, allowing for comparisons between individual school injury rates and providing a national perspective.

During a preparticipation examination, the parent of a high school freshman asks if the risk of injury is higher if his daughter plays soccer or volleyball. On what should you base your answer? A. Data in your school ' s annual sports medicine report specifi c to the number of soccer players and volleyball players treated by your athletic training staff during the previous two seasons. B. Data from the National High School Sports-Related Injury Surveillance Study specifi c to athlete injury exposure rates for the sports of soccer and volleyball. C. Data from the National Center for Catastrophic Sports Injury Research specifi c to the number of catastrophic injuries in high school soccer and volleyball. D. Data from the National Safety Council specifi c to high school volleyball and soccer injury rates in your state.

B: The athlete in this scenario has lost more than 2% of his body weight, indicating he is severely dehydrated and at increased risk for injury as well as heat illness

During preseason football a sophomore records a third day prepractice weight of 193 lbs. His fi rst session prepractice weight was 200 lb. What actions should be taken in managing this athlete? A. The athlete should be encouraged to drink fl uids before and while participating fully in the practice session. B. The athlete should be encouraged to drink fl uids and be held out of practice until he returns to his normal body weight. C. The athlete should be encouraged to drink fl uids and should be restricted from full-contact drills during practice. D. The athlete should be encouraged to drink fl uids and be allowed to practice until he demonstrates or experiences signs and symptoms of exertional heat illness.

A: Urine color charts typically range from 1 to 8 with the extreme ends of the range indicating possible dangerous hydration issues

During preseason soccer training, you post urine color charts in the restrooms with specimen collection cups and request athletes to report their urine color after practice. An athlete reports a urine color of 4 based on the chart pictured. Based on these fi ndings, how would you educate your athlete? A. You are slightly dehydrated and should consciously consume a few extra cups of water in the next few hours after practice and over the next 24 hours. B. You are very well hydrated and should follow your normal hydration plan, but be sure you are also getting enough sodium and electrolytes. C. You are very dehydrated, and we need to review your current hydration plan to determine where additional fl uids can be added. D. You are extremely dehydrated and need to be referred for further evaluation.

B: Assessment data can be used to decrease injuries by implementing proactive measures to address defi ciencies.

During your preparticipation screening, you determine that 75% of the athletes on the baseball team have sit and reach scores below normal limits. How would you best use this information? A. Instruct the athletes to work on improving their fl exibility. B. Work with the coaches to incorporate fl exibility exercises into the team warm-up and cool-down. C. Meet with the strength and conditioning coach to revise the weight training program. D. Ask the team nutritionist to conduct a special seminar on hydration and electrolytes.

A: Due to the chemical makeup of vomitus, bloodborne pathogen viruses are not carried in this bodily fl uid

Exposure to which of the following body fl uids does not require the use of universal precautions? A. Vomitus B. Cerebrospinal fl uid C. Vaginal secretions D. Synovial fl uid

A: Open-cell foam allows air to transfer from cell to cell and deforms quickly to stress, making it a minimal shock absorber

Foam is commonly used for constructing padding. One type of foam, open-cell foam, can be used as a liner for custom-made pads. What statement is correct regarding open-cell foam? A. Deforms quickly when stressed, providing minimal shock absorption B. Deforms slowly when stressed, providing minimal shock absorption C. Deforms quickly when stressed, providing a high level of shock absorption D. Deforms slowly when stressed, providing a high level of shock absorption

C: A thumb spica semirigid cast limits fi rst metacarpophalangeal (MCP) joint and wrist range of motion. The distal end of the cast is applied proximal to the MCP joints of the second through fi fth digits and proximal to the interphalangeal joint of the thumb.

For which of the following injuries is a thumb spica semirigid cast indicated? A. Fifth distal interphalangeal joint sprain B. Third proximal interphalangeal joint sprain C. First metacarpophalangeal (MCP) joint sprain D. Second metacarphophalangeal (MCP) joint sprain

A: Athletes participating in National Collegiate Athletic Association (NCAA) and National Federation of High School Sports (NFHS) sanctioned practices and competitions in the sports of fencing, fi eld hockey, ice hockey, and boys/men ' s lacrosse must use protective padding for the elbow and/or forearm

For which of the following sports does the National Collegiate Athletic Association (NCAA) and the National Federation of High School Sports (NFHS) require athletes use protective padding for the elbow and/or forearm during all practices and competitions? A. Ice hockey B. Soccer C. Wrestling D. Basketball

A: Fluid consumption that exceeds urine output and results in increased body weight after an exercise session more than 1 hour in duration increases the risk of hyponatremia

How can physically active individuals exercising for more than an hour in duration easily monitor adherence to proper hydration protocols to decrease risk of hyponatremia? A. Monitor weight before and after activity B. Assess urine color using standard urine color chart C. Assess body temperature before and after activity and if any symptoms occur D. Monitor urine output compared with water intake over a 24-hour period

A: Assigning each helmet a code and noting date of purchase and subsequent actions taken will allow recertifi cation and reconditioning within the recommended time frame.

How might helmets be tracked to ensure appropriate inspections, reconditioning, recertifi cation, and disposal occur? A. Each helmet should be assigned a code number at the time of purchase and then tracked. B. All helmets must be inspected, reconditioned, and recertifi ed each year. C. Only helmets that were used during the season need to be inspected and recertifi ed. D. All helmets should be inspected annually, and any helmet that is damaged must be disposed of by following manufacturer ' s guidelines.

D: It is recommended that educational sessions occur twice during the season with the fi rst session including parents for high school students.

How often should formal educational sessions regarding the risk of head-down contact in football be conducted? A. Once before the fall season and once before the spring season B. Each time the team is assessed with a spearing penalty during the season C. Once a year before the start of the season D. Once before the start of the season and once midway through the season

B: Modality equipment should be inspected and calibrated annually to make sure that it meets national electrical code guidelines

How often should therapeutic modality equipment be inspected and calibrated to ensure patient safety? A. Every 6 months B. Annually C. Every other year D. Every 5 years

C: Patients should be encouraged to emphasize exhalation during exertion and avoid holding the breath to prevent the Valsalva maneuver from occurring

How should you instruct an athlete to breathe while performing a bench press? A. Inhale as the bar is thrust upward and exhale as the bar is lowered. B. Hold the breath as the bar is lowered and inhale as the bar is thrust upward. C. Inhale as the bar is lowered and exhale as the bar is thrust upward. D. Hold the breath until one repetition is complete. E. Inhale as the bar is lowered and hold the breath as the bar is thrust upward.

C: Intrinsic and extrinsic risk factors for hypothermia have been identifi ed through case studies and controlled cold water immersion studies. Extrinsic factors include environmental, cold water immersion, wet clothing, body type, age, gender, illness, and injury. Intrinsic factors include substance abuse, exercise intensity, physical fi tness, hypoglycemia, and time and place.

Hypothermia, which can result in long-term morbidity or death, can occur in organized sports and in physically active people. Athletic trainers should monitor both extrinsic and intrinsic factors that can predispose a person to hypothermia. Which of the following is an extrinsic risk factor for developing hypothermia? A. Hypoglycemia B. Exercising at a low intensity C. Exercising with whole or partial body immersed in cold water D. Fitness level falls below physical demands of exercise

C: Central nervous system dysfunction and a rectal temperature of 105°F are the exertional heatstroke diagnostic criteria signs and symptoms

In addition to a rectally assessed core body temperature above 105°F, what postcollapse symptom is the other criterion for diagnosing exertional heatstroke? A. Involuntary muscle spasm B. Tachycardia C. Central nervous system dysfunction D. Hyperventilation

B: Athletes often fail to record everything on their health history because they may not think it is pertinent information. Screening medications may help fi ll in some gaps

In addition to appropriate management and monitoring of medical conditions, why is it important that the examiner review all medications listed on the health history with the athlete during the preparticipation examination? A. The athlete may not be receiving appropriate medical care. B. The review of medications may provide information about underlying medical conditions that were not disclosed on the health history. C. The reviewer can screen for substance abuse issues. D. The medications must be documented for drug testing purposes.

A: According to the National Athletic Trainers' Association (NATA) position statement on lightning safety, if a thunderstorm appears imminent, activity should be suspended. If thunder can be heard, lightning is close enough to be a hazard, and all persons at the athletic event should seek a safe location immediately.

In the fi nal 5 minutes of a high school girls' lacrosse game the wind picks up, the skies darken slightly, and you hear thunder. Based on recommendations from the National Athletic Trainers' Association (NATA) position statement on lightning safety for athletics and recreation, what action should be taken? A. All individuals, including coaches, offi cials, players, and spectators, should immediately go to a safe location. B. The coaching staffs and offi cials should be informed of the possibility of lightning and instructed to begin making plans for eventual evacuation if lightning is spotted. C. The designated "weather watcher" should, based on the sound of the thunder, estimate the arrival time of the storm ' s lightning and inform all individuals, including coaches, offi cials, players, and spectators, of this estimated arrival time. D. The school ' s director of athletics or highestranking administrator in attendance should inform the coaching staffs and offi cials if the game must be completed, regardless of weather, if needed in determining seeding for postseason competition

D: Fully acclimatized individuals can exercise outdoors until wet bulb globe temperature (WGBT) levels reach red (86.1°F-89.9°F), whereas nonacclimatized individuals may be susceptible to heat illness at very low levels (<78.0°F).

In what way does level of acclimatization impact recommendations for activity as the wet bulb globe temperature (WBGT) index increases? A. Activity recommendations remain the same regardless of levels of acclimatization. B. As the WGBT index increases the recommended work/rest ratio decreases, but water intake remains static for acclimatized individuals. C. Water intake recommendations increase for unacclimatized individuals, but the work/rest ratio remains the same for both groups. D. Nonacclimatized individuals are at risk of heat illness with extreme exertion even at low WGBT levels (<78.0°F).

A: Athletes with poorly controlled seizure disorders can safely participate in moderately strenuous noncontact sports that pose no life-threatening risk to self or others should a seizure occur

In which of the following sports activities could an athlete with poorly controlled epilepsy safely participate? A. Ballet B. Swimming C. Archery D. Volleyball

C: While there are currently six vaccines available to prevent meningococcal conditions, athletes may still contract (and spread) meningitis. To minimize the risk of this infectious disease spreading, it is recommended that anyone who has had face-to-face contact with a person diagnosed with meningitis begin an appropriate prophylactic medication.

Last night a member of your school ' s softball team was hospitalized with the diagnosis of bacterial meningitis. What action should be taken to minimize the risk of this illness spreading? A. Quarantine all softball players for at least 36 hours. B. Refer all softball players and persons who had face-to-face contact with the sick athlete for a cerebrospinal fl uid (CSF) examination, obtained through a lumbar puncture. C. Ensure that all persons who had face-to-face contact with the sick athlete begin prophylactic medication, such as ciprofl oxacin. D. Refer all softball players and persons who had face-to-face contact with the sick athlete for a complete blood count (CBC) and urinalysis.

D: The four types of credentialing laws that regulate the practice of athletic training are certifi cation, registration, exemption, and licensure. Licensure laws limit the practice of athletic training to persons who have met the requirement of a licensing board, usually by passing a licensing examination administered by the board.

Most states regulate the practice of athletic trainers for the protection of the public as well as for the advancement of the profession. Which form of governmental credentialing law is considered the most restrictive? A. Certifi cation B. Registration C. Exemption D. Licensure

B: When fi tting a mouth guard, submerge the guard in boiling water until pliable, approximately 20 to 30 seconds, and avoid biting the guard; instead, just close the mouth and suck the guard against the upper arch of the mouth. Cutting or trimming the mouth guard may minimize the protection provided

Mouth-formed mouth guards are one category of mouth guards. What action should be followed when fi tting a mouth-formed mouth guard? A. Submerge the mouth guard in boiling water for at least 5 minutes. B. Place the heated guard into the mouth, press the lips together to create a seal, and suck the guard against the upper arch. C. With the heated guard in the mouth, bring the upper and lower jaw together to gently bite into the guard to imprint teeth pattern. D. After the mouth guard is formed, cut the posterior ends and trim the edges next to the gums as much as needed for comfort.

B: While there is no perfect choice for clothing, research supports selecting clothing that is breathable, will not hold moisture, is loose fi tting, and dries easily. The National Athletic Trainers' Association (NATA) position statement on environmental cold injuries recommends using three layers of clothing whereby the inner layer against the skin can wick away sweat from skin; the middle layer provides warmth and insulation; and the outer layer is breathable, allowing moisture to evaporate while protecting against cold air, wind, and rain

On a cold spring afternoon, you assess the weather conditions. Based on air temperature and wind speed you determine the lacrosse players practicing on the turf fi eld are at risk for hypothermia. What clothing guidelines should you recommend the players follow to minimize their risk of hypothermia? A. Wear snug, tight-fi tting clothing next to the skin. B. Wear clothing next to skin that does not hold moisture. C. Wear nonbreathable clothing as outermost layer. D. Wear tight-fi tting, moisture-absorbing socks and gloves.

A: The spotter has three functions: to protect the lifter from injury, to make recommendations on proper lifting technique, and to help motivate the lifter

One function of a spotter for a person lifting free weights is to protect the lifter from injury. Before each exercise, what should the lifter communicate to the spotter to best protect himself from injury? A. The number of repetitions the lifter plans to complete B. Where the spotter should stand C. The weight achieved during the most recent 1-RM (repetition maximum) test D. The amount of desired verbal motivation

B: Increased weight bearing and increased calcium intake enhance bone strength, therefore decreasing the risk of developing osteoporosis and bone weakness. Weight-bearing activities apply stresses to the bones, resulting in an increase in compressive load tolerance.

Osteoporosis is a condition that predominantly affects older women. Which of the following prevention strategies would provide the most signifi cant impact? A. Moderate swimming and increased vitamin C intake B. Weight-bearing activities and increased calcium intake C. Maintaining 10% body fat and using a minimal resistance stationary bike D. Increased electrolytes and use of nonsteroidal antiinflammatory medication E. Avoidance of physical activity and dairy products

D: A major depressive episode involves experiencing depressive symptoms for at least 2 weeks, including a lack of interest in usual events and a general depressed mood along with at least four of the following: appetite changes (with weight loss or gain), insomnia or hypersomnia, psychomotor agitation or retardation, fatigue, feelings of worthlessness or guilt, diffi culty concentrating or indecisiveness, or recurrent thoughts of death or suicide

Over the past 3 weeks, you have noticed alarming changes in an athlete. He is generally uninterested in training sessions and matches, and he appears unhappy, fatigued, and unfocused. Additionally, he appears to have lost weight and looks as though he is not sleeping enough. Which of the following psychological conditions should you suspect? A. Posttraumatic stress disorder B. Obsessive-compulsive disorder C. Generalized anxiety episode D. Major depressive episode

B: Overtraining can occur when athletes or patients overemphasize resistance training and do not allow for suffi cient periods of rest and muscle recovery. The use of proper technique, adequate rest, and proper nutrition can aid in preventing athletes and patients from developing the negative consequences of overtraining

Overtraining can result in physical fatigue, decreased muscle strength, decreased physical performance, and psychological breakdown resulting in a lessened desire to train. In what situation is overtraining likely to occur? A. Training sessions that emphasize proper technique B. Training sessions that exceed the body ' s physiological limits C. Training sessions that emphasize cardiovascular training over strength training D. Training session volume exceeding that of the previous training sessions

D: To maintain a safe environment, electrical equipment should comply to National Electrical Code guidelines

Practicing electrical safety in an athletic training facility is important to prevent injuries to patients and therapists. Which action provides the best legal protection from a potential lawsuit? A. To ensure grounding, use only three-pronged wall outlets. B. Defective equipment should be labeled and used only intermittently. C. Remove plugs by pulling cord at an angle perpendicular to the outlet. D. Evaluate electrical equipment yearly, ensuring compliance to National Electrical Code guidelines.

E: The National Collegiate Athletic Association (NCAA) and the National Association of State High School Associations have established participation regulations for wrestlers with bacterial, viral, and fungal infections.

Preparticipation skin checks for wrestlers determine participation eligibility. With which of the following dermatological conditions may a wrestler compete, provided that the active lesions are adequately covered? A. Community-acquired methicillin-resistant Staphylococcus aureus (CA-MRSA) B. Herpes simplex virus C. Impetigo D. Furuncles E. Tinea corporis

B: In most cases, both intrinsic and extrinsic risk factors for exertional heatstroke can be addressed to reduce episodes of exertional heatstroke. Other intrinsic risk factors include sleep loss, dehydration, overzealousness, high muscle mass-to-body fat ratio, presence of a fever, use of certain medications, and some skin disorders.

Recognizing and addressing intrinsic risk factors for exertional heatstroke is an important prevention strategy. Which of the following is an intrinsic risk factor for exertional heat stroke? A. Athletic equipment B. Inadequate heat acclimatization C. Lack of education and awareness of heat illnesses by coaches, athletes, and medical staff D. High ambient temperature, solar radiation, and humidity

B: Spearing is the action of intentionally making a tackle with the head down so that the helmet strikes fi rst. This places an axial load on the cervical spine leaving the athlete at risk for cervical fracture, spinal cord injury, or death. It also increases the risk of concussion and other head injury. This rule was implemented to attempt to reduce these injuries.

Rules have been instituted in some sports to minimize the risk of injury to participants. In 1976 the practice of spearing was deemed illegal. What action constitutes spearing? A. During a tackle, two football players collide and, just before they hit the ground, their face masks make contact. B. A defensive back tackles a receiver, making initial contact with his helmet. C. A defensive lineman sacks the quarterback and, as they hit the ground, the top of the lineman ' s helmet contacts the face mask of the quarterback. D. A running back jumps over two downed players on the goal line and lands on the top of his head in the end zone. E. A running back running down the sideline gets wrapped up by a linebacker, and as he turns away to escape the tackle, the top of his helmet hits the face mask of the defensive back.

D: Sickle cell trait (SCT) is common in people who descend from areas where malaria is common, such as parts of Africa, so SCT should be thought of as malarial, not racial. SCT occurs most commonly in African Americans but also occurs in persons of Hispanic ethnicity and persons from the Mediterranean, the Middle East, and parts of India.

Sickle cell trait (SCT) is a condition that comes from inheriting one gene for normal hemoglobin and one gene for sickle hemoglobin and can be present in persons of any ethnicity. In which population group is the incidence of SCT most common? A. Eastern European B. Middle Eastern C. Hispanic D. African American

B: A sunscreen with SPF 15 indicates the athlete can be exposed to the sun ' s ultraviolet (UV) light 15 times longer than without any sunscreen before the skin begins to turn red. It also means it must be applied twice as often as would be necessary with a sunscreen with SPF 30

Some members of a track team residing in one of the northern states have expressed concern with getting sunburn on their upcoming southern spring break training trip. What information should be provided to these athletes regarding the use of sunscreen to prevent sunburn? A. Sunscreen is not needed by athletes with dark complexions. B. Sunscreen with a higher number SPF provides a longer period of protection against the sun ' s ultraviolet (UV) rays. C. Sunscreen needs to be used only for practice sessions occurring between 10 a.m. and 4 p.m. D. Sunscreen should be applied 15 to 20 minutes after sun exposure.

A: Sports are classifi ed according to the risk of injuries occurring; these classifi cations include collision, limited contact, or noncontact. The American Academy of Pediatrics (AAP) classifi es the following sports as contact or collision sports: basketball, boxing, diving, fi eld hockey, tackle football, ice hockey, lacrosse, martial arts, rodeo, rugby, ski jumping, soccer, team handball, water polo, and wrestling

Sports can be classifi ed based on their comparative risk of injury. According to the American Academy of Pediatrics (AAP), in which of the following categories would soccer be classifi ed? A. Contact/collision B. Noncontact C. Intermittent contact D. Noncollision E. Limited contact

C: Because of the Americans With Disabilities Act of 1990, the athlete has the power to make the fi nal decision regarding his participation

Sports medicine physicians and health-care professionals may recommend an athlete with certain medical conditions or injuries discontinue or not commence participation in certain sports or physical activity. What person can make the fi nal decision regarding disqualifi cation of an athlete? A. Team physician B. Athletic director C. Athlete D. Athletic trainer

D: By using correct technique, injury risk can be minimized, and the client can gain the most from each exercise

Strength and conditioning programs often incorporate plyometric exercises to develop muscular power. What is the most important consideration when adding plyometric exercises to a fi tness program? A. Client ' s muscular strength B. Client ' s speed of movement, quickness C. Client ' s total body weight D. Client ' s ability to properly execute each exercise

A: Medical history questions are used as initial screening tools, but a true evaluation must be conducted before referral is made for treatment or monitoring

The medical history portion of your preparticipation physical examination (PPE) asks several screening questions related to risk factors for mental health concerns. What actions should be taken for an athlete who answers "yes" to any of these questions? A. The physician should have a private conversation with the athlete and determine if referral is indicated. B. The student athlete ' s parents should be notifi ed of your concerns. C. The student athlete should be referred to the school counselor. D. The student athlete should be monitored during the season and questioned periodically by the athletic trainer for warning signs.

A: Wet bulb globe temperature (WBGT) is calculated from the dry bulb temperature (DBT), which measures the ambient air temperature; the wet bulb temperature (WBT), which measures the humidity; and the globe temperature (GT), which measures the sun ' s radiation. The following formula is used to calculate WBGT: WBGT = (0.1 degree X DBT) + (0.7 degree X WBT) + (GT X 0.2).

The universal wet bulb globe temperature (WBGT) index provides objective information athletic trainers can use in making recommendations regarding exercise in hot and humid environments. WBGT is calculated from three different thermometers providing three environmental variables. Which thermometer provides a measure of the sun ' s radiant energy? A. Globe temperature (GT) B. Wet bulb temperature (WBT) C. Dry bulb temperature (DBT) D. Sol bulb temperature (SBT)

A: SPF stands for sun protection factor. The number after SPF, such as 15, means the athlete with SPF 15 protection can stay in the sun 15 times longer than without protection before he or she starts to get sunburnt

The volleyball team is participating in a beach volleyball tournament. The coach has given the players sunscreen with SPF 15. What does an SPF of 15 imply? A. The athletes can be exposed to ultraviolet (UV) light 15 times longer than they could without any sunscreen before their skin turns red. B. The athletes can safely stay in the sun for 150 minutes before their skin turns red. C. The athletes have 15 microlayers of skin protection against harmful UV rays. D. The athletes will have protection against 15 different types of UV rays. E. The athletes can safely stay in the sun for 15 hours before their skin turns red

B: Based on the Oxford Centre for Evidence-Based Medicine model, systematic reviews of randomized controlled trials provide the strongest evidence

The women ' s soccer coach e-mailed the director of athletic training requesting an anterior cruciate ligament (ACL) injury prevention program be developed and administered to her players during the upcoming nontraditional season. As you are the newly hired intern, you have been tasked with researching the current literature on ACL injury prevention programs. Which type of published study can provide you the strongest level of evidence? A. Randomized controlled trial (RCT) B. Systematic review of RCTs C. Outcomes research D. Case series

D: Low-calorie foods such as celery, grapefruit, and apples should be included in a healthy weight loss plan, but the body will not expend more calories digesting and absorbing the food than the caloric content of the food

The wrestling coach is speaking with his wrestlers about weight loss. He instructs his wrestlers to eat as much celery and apples as they want because these foods are catabolic foods, which promote weight loss. He informs the wrestlers that these foods contain fewer calories than are required to chew, digest, and absorb them. How should you respond to this coach ' s instructions? A. Confi rm the coach ' s information is accurate and note that the wrestlers will benefi t from the increased intake of fruits and vegetables. B. Confi rm the coach ' s information is accurate, but encourage the wrestlers to also eat as much grapefruit as possible, as grapefruit is the strongest negative calorie food. C. Inform the coach and the wrestlers that metabolism is signifi cantly increased with eating so any fruits and vegetables should aid in weight loss. D. Inform the coach and the wrestlers that while metabolism increases with eating, the increase is small, and there is no research in support of any catabolic foods whereby more calories are used to metabolize a food than the number of calories in the food.

D: The maintenance and fi tting of protective equipment is critical to minimizing risk of sports injuries, particularly in collision sports such as football. Less skilled players with a lower fi tness level are at increased risk for injury, and providing these players with worn-out equipment that may not fi t properly compounds their risk for injury

This fall your high school has a record number of boys wanting to play football. The returning varsity players and almost all of the junior varsity players have received properly fi tted new or reconditioned equipment. The equipment supply has been exhausted, but the coaches want to give all the remaining boys without equipment a chance to play, so they inquire about using the old, worn-out equipment in the storage room, even if it just "almost fi ts." What is the most important information that should be communicated to the coaching staff regarding this stored equipment? A. The school emblem on these old worn-out helmets is not current and so will not match the other helmets. B. The screws holding the face masks on the old helmets are mostly rusted and stripped, not allowing for changing of face masks to ensure position-specifi c face masks. C. These helmets were stored in a moist and dark storage room, so they will likely have mold and other fungi on the inner surface and padding. D. Junior varsity players are less skilled and experienced, so wearing worn-out and improperly fi tted equipment increases their risk of injury.

A: In most cases, intrinsic risk factors can be controlled, whereas extrinsic factors are generally outside the control of the athlete.

To avoid catastrophic cervical spine injury in sports, athletic trainers should attempt to control for both extrinsic and intrinsic predisposing risk factors. Which of the following is an intrinsic risk factor for cervical spine injury in sports? A. Improper technique, such as leading with the head into contact B. Offi cials not calling penalties for illegal play known to cause cervical spine injury C. Lack of education and awareness of causes of cervical spine injury among coaches when instructing athletes D. Faulty athletic equipment

C: Research supports the use of functional performance testing as a component of the returnto- play decision-making for an athlete who has sustained an ankle sprain. Performance of the previously injured limb should be within 80% of the performance of the contralateral side

To determine an athlete ' s readiness to return to participation following a 3-week rehabilitation program for a lateral ankle sprain, you use the Star Excursion Balance Test and a single-legged hop for distance test. To minimize this athlete ' s risk of injury on return to sport-specifi c tasks, what percentage of the uninjured limb ' s performance should be demonstrated by the injured limb? A. 50% B. 60% C. 80% D. 100%

B: Sensitivity is the value used to describe a test ' s ability to detect the disorder when it is present and is also known as the "true positive rate.

To determine the diagnostic accuracy of a newly designed physical examination test, researchers calculate the test ' s sensitivity. Which statement regarding test sensitivity is correct? A. Sensitivity is also known as the "true negative rate." B. Sensitivity is calculated as true positives/(true positives + false negatives). C. Sensitivity describes the test ' s ability to detect patients who do not have the disorder. D. Sensitivity is calculated as true negatives/(true negatives + false positives). E. Sensitivity is also known as the "diagnostic gold standard."

A: The National Athletic Trainers' Association (NATA) position statement on skin diseases recommends selecting a disinfectant or detergent that is registered by the Environmental Protection Agency and to use the product by following the manufacturer ' s instructions regarding amount, dilution, and contact time. Failure to follow manufacturer ' s instructions, rather than the type of cleaning agent, has been cited as the primary reason for facility pathogen accumulations.

To minimize disease transmission, equipment and surfaces in sport venues and health-care facilities should be frequently and routinely cleaned. What should be the primary consideration when selecting and using a disinfectant or detergent for routine cleaning and disinfecting? A. All the manufacturer ' s recommendations for amount, dilution, and contact time are followed B. The product is endorsed by the U.S. Centers for Disease Control and Prevention C. The product contains trimethoprim with sulfamethoxazole (TMP-SMZ), clindamycin, or linezolid D. The manufacturer ' s published cost/benefi t ratio

A: The repetitive nature of using a keyboard at a computer workstation can lead to overuse injuries. Proper worksite ergonomics can aid in minimizing the strain from performing repetitive tasks. Workers seated at computer stations should select a chair that allows for a 2-in. space between the posterior knee and the chair when seated at the back of the chair. The chair armrests should be situated such that relaxed arms fall on them without abducting the shoulder, and the back of the chair should be no lower than the inferior angle of the scapulas.

To minimize musculoskeletal overuse conditions in the workplace, what is the proper position to assume when working at a computer? A. Adjust chair seat height so that when seated all the way back in the chair the worker ' s feet are fl at on the fl oor with the hips and knees each fl exed to 90°. B. Chair size should allow the worker to sit in the chair with hips all the way to the back, creating no space between the chair and the posterior knee. C. While seated the worker ' s relaxed arms should be abducted approximately 45° to rest on the chair arm rests. D. The back of the chair should be no higher than the inferior angle of the scapulas.

B: Cold temperatures, wind, and dampness can predispose athletes to hypothermia

To prevent hypothermia, coaches and athletes should be educated on assessing environmental conditions. In addition to air temperature, what environmental factor should coaches and athletes consider when determining athletes' risk for hypothermia? A. Pollen count B. Wind speed C. Dew point D. Radiant energy of sun

D: SAID is an acronym for specific adaptation to imposed demand and states that when the body is subjected to stress and overloads of varying intensities, it will gradually adapt.

To which principle of conditioning does the SAID principle directly apply? A. Individuality B. Consistency C. Progression D. Overload E. Safety

B: This response is consistent with recommendations for industry standards regarding helmet recertifi cation and reconditioning

Ultimately the frequency of helmet recertifi cation and reconditioning is up the discretion of the athletic trainer. However, if no warranty exists or after the warranty expires, how often must helmets be recertifi ed and reconditioned using a vendor approved by the National Operating Committee on Standards for Athletic Equipment (NOCSAE)? A. Every year B. Every 2 years C. At the end of the playing season D. Every 4 years E. Recertifi cation/reconditioning not recommended after warranty expires

B: Sharps containers are for razor blades, scalpels, and needles that create a cutting risk. Scissors and tweezers should be sterilized using a disinfecting agent. Uniforms containing blood should be evaluated for infectivity, and contaminated towels and other linens should be washed in hot water (71°C [159.8°F]) for 25 minutes.

Universal precautions aid medical and allied health-care professionals in minimizing exposure to bloodborne pathogens. Which of the following demonstrates appropriate application of universal precautions in the athletic training setting? A. Blood-covered scissors should be disposed of in a sharps container, not sterilized. B. Contaminated surfaces should be cleaned with a solution consisting of 1 part bleach to 10 parts water or with a disinfectant approved by the Environmental Protection Agency. C. Uniforms containing blood should be removed and changed before returning the athlete to competition. D. Blood-soaked towels and other linens should be washed in cold water to prevent the blood from setting into the material.

B: Nosocomial infections are infections a patient gets while seeking care at a medical facility for a different condition. Staphylococcus aureus is a bacterial infection transmitted through direct contact, either person to person or person to surface, and can be contracted at a medical or athletic training facility.

Washing with soap and water is a proper hygiene practice effective in minimizing a patient ' s risk of acquiring a nosocomial infection. Which of the following infectious diseases is a patient at risk of acquiring in a medical or athletic training facility? A. Zika virus B. Staphylococcus aureus C. Mononucleosis D. Chlamydi

C: The National Collegiate Athletic Association (NCAA) Sports Medicine Handbook recommends avoiding the use of rubberized clothing and suits

Wearing which clothing has the greatest potential to increase an exercising athlete ' s risk for heat illness? A. Cotton short-sleeve T-shirt B. Net jersey C. Rubberized long-sleeve pullover D. Cotton long-sleeve T-shirt

C: Athletic trainers should comply with all state and federal laws specifi c to medication storage and management. All medications should be stored securely to prevent unauthorized use.

What are the recommendations for storage of prescription and over-the-counter (OTC) medications in the athletic training room? A. Prescription medications should not be stored in an athletic training room, but OTC medications can be safely stored in a storage closet or cabinet. B. Prescription and OTC medications must be stored in a locked closet that is accessible only to the team physician. C. Prescription and OTC medications should be stored in a climate-controlled locked cabinet with tamperproof locks accessible only to authorized personnel, including the team physician and certifi ed athletic trainer. D. Prescription medication must be stored in a locked cabinet that is accessible only to the certifi ed athletic trainer and team physician, whereas OTC medications can be freely accessible to staff and athletes.

B: The Dietary Guidelines of Americans document serves as a blueprint for developing eating plans for most Americans.

What document published every 5 years by the U.S. Department of Agriculture and the Department of Health and Human Services provides Americans advice on good eating habits to promote health? A. FDA Recommendations for Americans B. Dietary Guidelines for Americans C. MyPlate for Americans D. Dietary Reference Intakes for Americans

D: Predisposing factors for an exertional sickling episode include altitude, heat, dehydration, asthma, and other illnesses

What environmental condition puts an athlete with sickle cell trait at risk for an exertional sickling episode? A. Low air temperature B. High pollen count C. High wind speed D. High altitude

B: Dampness or wetness further increases risk of hypothermia for an athlete exercising in a cold and windy environment

What environmental condition, when combined with low air temperatures and wind, creates an environment that can predispose an athlete to hypothermia? A. Lightning B. Wetness C. Cloudiness D. High pollen count

D: Sound electrical safety parameters include not touching electrical devices while in contact with water.

What important instruction should be posted over the whirlpool and given to the patient verbally before initiating treatment to reduce the risk of electrical shock? A. No horseplay in the treatment area B. No jewelry may be worn in whirlpool C. Wash the treatment area with soap and water before immersion to remove sweat and other electrolyte particles D. Do not turn the machine on or off while immersed in the water

C: Excess dietary vitamin D can result in hypercalcemia, which can cause mineral deposits and calcifi cation in blood vessels, kidneys, and other soft tissues

What is considered a cause of mineral deposits in the kidneys, blood vessels, and other soft tissues in the body? A. Excessive dietary vitamin A B. Inadequate dietary vitamin D C. Excessive dietary vitamin D D. Excessive dietary vitamin K E. Inadequate dietary vitamin K

C: The feet increase slightly in size over the course of the day, so fi tting late in the day will ensure better fi t.

What is the best time of day to fi t an athlete for shoes? A. First thing in the morning B. After a workout C. Late in the day D. After completion of a warm-up and stretching session

D: Spread most commonly through contact with the blood of an infected person, hepatitis C virus (HCV) has extremely high risks of chronic infection, with 80% of persons infected not demonstrating any signs or symptoms

What is the most common chronic bloodborne pathogen infection in the United States? A. Human immunodefi ciency virus (HIV) B. Methicillin-resistant Staphylococcus aureus (MRSA) C. Hepatitis B virus (HBV) D. Hepatitis C virus (HCV)

B: Bile is produced by the liver and then stored in the gallbladder. The gallbladder holds the bile until it is needed, then the gallbladder pushes it into the common bile duct and into the small intestine

What is the primary function of the gallbladder? A. Secrete digestive enzymes B. Store bile C. Release insulin into the bloodstream D. Produce hormones for regulating carbohydrate metabolism E. Store glucose

A: An ergonomic risk assessment (ERA) focuses on identifying specifi c factors about a job that increase the likelihood that someone will be injured. The ERA is compared with injury rates to establish priorities for modifying tasks to create a safer and more effi cient work environment.

What is the purpose of conducting an ergonomic risk assessment (ERA) and comparing it with injury statistics? A. To identify the primary physical factors of a job that might be changed to minimize the risk of injury B. To provide feedback to an employee about work performance C. To modify current workers' compensation standards D. To improve effi ciency and productivity

B: Assumption of risk statements demonstrates that the athlete understands participation may lead to injury and provides some protection from legal action if the athletic training staff is not negligent in their actions.

What is the purpose of having an athlete sign an assumption of risk statement? A. To excuse the athletic trainer and coaches from responsibility when injury occurs B. To acknowledge that participation in sports carries some inherent risk of injury and reinforce that the athlete is willing to take on that risk C. To absolve the athletic department from any liability if the athlete is injured D. To indicate understanding on the part of the athlete that equipment manufacturers cannot be held responsible in the event of an injury

B: Wellness screening includes questions about diet, rest, exercise, stress management, and weight control as well as alcohol, drug, and tobacco use.

What is the purpose of including a wellness screening as part of the preparticipation examination? A. To disqualify the athlete from participation B. To determine if the athlete is engaging in healthy or unhealthy behaviors and lifestyle habits that pose a threat to wellness C. To screen for health maintenance, personal hygiene, and potential for infectious diseases D. To assess physical maturity in order to protect young, physically undeveloped athletes

A: Healthy People is a set of more than 1,200 evidence-based objectives and goals designed to improve the health of the nation. It is updated every 10 years and is developed by representatives from multiple federal agencies

What is the purpose of the Healthy People initiative that is updated every 10 years? A. To provide comprehensive goals and objectives designed to improve the health of the nation B. To provide a census of the current health status of the nation C. To compare the health of our nation with that of other nations worldwide D. To compare the health of individuals in different states and allocate federal funding accordingly

C: Exercising at 70% of maximum heart rate is necessary to see minimal improvement, and most trained individuals can comfortably sustain work at 85% of their maximum heart rate

What is the recommended intensity for cardiovascular training during a continuous workout? A. 30%-45% of maximum heart rate B. 45%-60% of maximum heart rate C. 70%-85% of maximum heart rate D. 90%-100% of maximum heart rate

C: Student athletes are responsible for making sure their equipment fi ts properly and can provide full protection as designed

What is the role of the student athlete in regard to protective equipment? A. The student athlete must wear and use the equipment properly. B. The student athlete must make sure his or her equipment is reconditioned based on recommended time frames. C. The student athlete must not modify the equipment in any way and should notify coaches, athletic trainers, or equipment managers if the equipment needs maintenance. D. The student athlete should follow the directions given by coaches and equipment managers.

D: Ear guards with a hard shell constructed of various nylon materials and with an inner padding are held onto the head using adjustable straps. Some designs are available with chin straps. Ear guards are the only wrestling equipment mandated by the National Collegiate Athletic Association (NCAA) and National Federation of High School Sports (NFHS).

What mandatory equipment is required by both the National Collegiate Athletic Association (NCAA) and the National Federation of High School Sports (NFHS) for any athlete participating in the sport of wrestling? A. Elbow pads B. Knee pads C. Mouth guard D. Ear guard

A: The Health Insurance Portability and Accountability Act (HIPAA) regulates how medical, healthcare, and allied health professionals share private health information about patients

What regulation guarantees that intercollegiate athletes have access to all of their medical records, while giving them more control over how their protected health information is used and disclosed? A. Health Insurance Portability and Accountability Act B. Family Educational Rights and Privacy Act C. Americans With Disabilities Act D. Medical Records Retention Act

C: Athlete exposure is calculated by multiplying the number of athletes at each practice/game by the number of practices/games. Exposure rate is determined by looking at the number of injuries during a set period of time.

What should you calculate if you want to determine the incidence of injuries for last season ' s men ' s soccer team, per the number of individual athlete exposures? A. Incidence of injury B. Injury prevalence C. Injury exposure rate D. Injury likelihood ratio

C: Closed-cell foam is heavier in weight allowing it to dissipate higher forces and rebound quickly

What type of material is capable of absorbing force through deforming its shape and then quickly returning to its original form? A. Open-cell foam B. Moleskin C. Closed-cell foam D. Felt E. Thermo-moldable plastic

D: Deeper grooves allow for grip on uneven surfaces

What type of running shoe tread is recommended by shoe manufacturers? A. Flat with herringbone pattern B. Pivot dot pattern C. Slip resistant D. Deep groove

C: Helmet testing to meet National Operating Committee on Standards for Athletic Equipment (NOCSAE) standards is based on the protective roles of the helmet. Whereas baseball batters ' helmets must undergo a projectile impact test to meet NOCSAE standards, baseball catchers ' helmets with face guards must undergo the drop test and the projectile impact test; lacrosse helmets must undergo a helmet stability and retention test, a drop method test, and a projectile impact test

What type of testing must a new baseball batter ' s helmet undergo to meet the National Operating Committee on Standards for Athletic Equipment (NOCSAE) standard? A. Stability and retention test B. Drop method test C. Projectile impact test D. Force dispersion test

E. Risk of participating in a sport is assumed when the athlete knows of and understands the dangers of that sport but voluntarily chooses to be exposed to those potential dangers. Assumption of risk can be verifi ed via a signed waiver or simply implied from the athlete ' s conduct while participating in the sport

When an adult chooses to participate in a sport, the knowledge of potential risks is in most cases clearly understood. Should an injury situation arise, when would the injured adult athlete assume none of the risk? A. When the athlete does not fully comprehend the assumption of risk policy B. When the athlete has never played the sport before the season in which the injury occurs C. When the athlete makes no effort to determine hazards associated with the sport D. When the athlete chooses not to sign the assumption of risk statement E. When the athlete is not warned of the dangers associated with participation

B: Due to repetitive throwing motion, upper extremity athletes maintain a normal arc of motion by exchanging excessive external rotation for a decrease in internal rotation.

When conducting a preparticipation examination, which of the following fi ndings would be considered a normal physiological adaptation to exercise and sport participation? A. Resting heart rate above normal range B. Increased external rotation and decreased internal rotation in the dominant arm of a baseball pitcher C. Resting blood pressure below normal values D. Quadriceps-to-hamstring ratio closer to 1

A: When designing an athletic training facility, at least nine design elements should be considered so that the facility is accessible by all, safe for clients and employees, and effi cient. These elements include size, location, ergonomics, electrical system, plumbing system, ventilation, lighting, specialized function areas, and accessibility

When designing a hydrotherapy area, which design elements are recommended to reduce the risk of patient injury? A. Electrical outlets must be at least 3 ft off the fl oor and equipped with ground fault circuit interruptors (GFCIs), and hydrotherapy tanks should be connected directly to a fl oor drain. B. Electrical outlets must be at least 2 ft off the fl oor, hydrotherapy tanks should be connected directly to a fl oor drain, and fl oors should be carpeted to reduce slippage. C. Electrical outlets must be equipped with GFCIs, fl oors should be sloped toward the drains to minimize puddling, and lighting should be a minimum of 140 W. D. The area should be a minimum of 60 sq ft per athlete being serviced, walls should be constructed from cinderblock, and fl oors should be carpeted to reduce slippage. E. The area should use only natural lighting and should be a minimum of 10 sq ft per athlete being serviced, and plumbing fi xtures should include mixing valves and foot pedal activators.

A: In addition to instruction in proper technique and close supervision, adolescent athletes should be encouraged to begin training with functional body weight exercises based on their level of maturation.

When developing a strength training program for adolescent athletes, what type of exercise is most appropriate in the initial stages of training? A. Body weight exercises B. Isokinetic exercises C. Plyometric exercises D. Free-weight exercises

C: When sweat or moisture on the skin evaporates and is converted to a gaseous state, the heat is transferred and removed

When exercising in an environment where ambient temperature is greater than skin temperature and relative humidity is low, what form of heat transfer is the body ' s most effi cient and most relied on means of heat loss? A. Radiation B. Convection C. Evaporation D. Conduction

C: A commercial shoulder arm sling can be used for forearm support when the shoulder complex, elbow, forearm, or wrist is injured. The sling should be applied with the elbow flexed such that the wrist and hand are superior to the elbow joint

When fitting a patient with a commercial shoulder arm sling, in approximately what degree angle should the elbow be positioned? A. 20° B. 45° C. 70° D. 140°

C: When fi tting a football helmet, ensure cheek pads are snug to cheeks, helmet covers base of skull, ear holes align with ears, helmet does not shift around when forces are applied, chin strap is centered, and you can measure two fi nger widths above eyes and three fi nger widths from athlete ' s face to face mask

When initially fitting or checking the fitting of an issued football helmet, the manufacturer ' s directions for proper fi t should be followed carefully. Which of the following represents a correct step in fitting a football helmet? A. Helmet ear holes should align with the inferior aspect of the athlete ' s ear lobes. B. Two finger widths should be available between the helmet cheek pads and the athlete ' s cheeks. C. The back of the helmet covers the base of the athlete ' s skull. D. The forehead pad of the helmet is at least three finger widths above the eyes.

D: First morning urine void is the most accurate, as checks throughout the day may be impacted by food and fl uid intake or exercise

When using a urine specifi c gravity measurement to determine hydration status, which urine void will give the most accurate information? A. First void following breakfast B. Void immediately after exercise C. Void immediately before exercise D. First void of the morning after waking up

A: The wet bulb globe temperature device uses three different tools to assess outdoor conditions. First, the dry bulb temperature is taken with a normal mercury thermometer. Next, the wet bulb temperature is taken by putting a wet piece of gauze around the end of a thermometer and swinging the thermometer around in the air to detect humidity. Finally, the globe temperature is taken with black metal casing around the end of a thermometer to assess radiation from the sun in the environment

When using a wet bulb globe temperature device, what factors are assessed to determine relative heat stress? A. Air temperature, humidity, and radiant heat in the local environment B. Air temperature and humidity C. Air temperature, humidity, and wind speed D. Air temperature, dew point, and radiant heat in the local environment E. Humidity and radiant heat in the local environment

E: The mission of the National Collegiate Athletic Association (NCAA) Committee on Competitive Safeguards Medical Aspects of Sports is to provide expertise and leadership to the membership to promote a healthy and safe environment for student athletes through research, education, collaboration, and policy development. This committee makes recommendations to modify safety guidelines, equipment standards, or sports rules of play. This committee also assists in development of the NCAA Sports Medicine Handbook content

Which National Collegiate Athletic Association (NCAA) committee is responsible for establishing policies and recommendations on safe sport participation? A. Committee on the Medical Aspects of Sports B. Committee on Sport Safety C. Committee on Risk Management in Intercollegiate Athletics D. Committee on Injury Surveillance and Prevention E. Committee on Competitive Safeguards and Medical Aspects of Sports

D: The likelihood of developing myocarditis is rare, but there are many potential causes, including viral, bacterial, and fungal infections as well as parasite infestations.

Which cardiac condition places ill athletes, particularly those who are febrile, at risk of sudden cardiac death? A. Hypertrophic cardiomyopathy B. Commotio cordis C. Marfan ' s syndrome D. Myocarditis

D: Exposure to dry temperatures well below freezing most often results in more severe cold injuries, such as frostbite and chilblain

Which cold injury is most likely to occur in an athlete exposed to a damp, freezing cold? A. Deep frostbite B. Superfi cial frostbite C. Chilblain D. Frost nip

A: Chilblains are the mildest and most superfi cial form of frostbite associated with vasoconstriction of peripheral vessels

Which condition is characterized by redness, tingling, swelling, and pain in the fi ngers or toes during exposure to cold, wet conditions over an extended period of time of 1 to 5 hours? A. Chilblains B. Frost nip C. Superfi cial frostbite D. Deep frostbite

D: Cantilevered shoulder pads have a strap that extends from front to back, causing the pads to arch above the tip of the shoulder. Because of the mechanics of this brace, the athlete who would benefi t most would be one who participates in a great deal of tackling or blocking due to the ability of the pads to dissipate the force effectively

Which football player would be most likely to wear a cantilevered shoulder pad? A. Quarterback B. Punter/kicker C. Youth football player D. Linebacker E. Wide receiver

E: The U.S. Food and Drug Administration Center for Devices and Radiological Health has approved low-level laser therapy for the treatment of carpal tunnel syndrome and shoulder and neck pain. This center also classifi es lasers by the accessible emission limit, which ranges from 1 to 4

Which government agency has developed national guidelines for the safety and effi cacy of low-level lasers? A. Occupational Health and Safety Administration B. Department of Health and Environmental Control C. American Medical Association Committee on Medical Safeguards D. U.S. Department of Health and Human Services E. U.S. Food and Drug Administration

C: Exertional heat exhaustion is caused by an inability of the body to sustain adequate cardiac output secondary to exercise-induced dehydration. Athletes with exertional heat exhaustion present with excessive thirst, fatigue, weight loss, low urine volume, weakness, incoordination, mental dullness, low urine volume, and slightly elevated body temperature

Which heat illness typically manifests with fatigue, excessive thirst, incoordination, mental dullness, and slightly elevated body temperature? A. Heat syncope B. Exercise-associated heat cramps C. Exertional heat exhaustion D. Exertional heatstroke

C: The corpus luteum (CL) is what is left of the follicle after a woman ovulates. This yellow hormone-secreting body is a dynamic endocrine gland that releases progesterone and small amounts of estrogen. The CL degenerates after a few days unless pregnancy has begun

Which hormone or hormones are secreted by the corpus luteum? A. Progesterone B. Human growth hormone (HGH) C. Progesterone and estrogen D. Luteinizing hormone (LH) E. Follicle-stimulating hormone (FSH)

A: Federal performance standards for ultrasound are regulated and mandated by the U.S. Food and Drug Administration (FDA).

Which is the only therapeutic modality for which federal performance standards currently exist? A. Ultrasound B. Electrical stimulation C. Phoresor D. Paraffi n bath E. Mechanical cervical traction

B: Whereas not as accurate as other screening tools, skinfold measurements are inexpensive, and accuracy can be improved with mastery of the measurement techniques

Which method of assessing body composition is relatively easy to perform clinically, requires limited equipment, and has a measurement error of only 3% to 5%? A. Bioelectric impedance B. Skinfold measurement C. Hydrostatic weighing D. Dual energy x-ray absorptiometry (DEXA) scan

B: An ergonomic assessment seeks to identify risk factors for injury so these can be addressed before injury occurs

Which of the following actions might be taken following the completion of a workplace ergonomic assessment? A. A list of employee requests for new equipment will be provided to the administrator. B. Recommendations for strengthening exercises, stretches. and rest breaks will be provided to the employee. C. A comprehensive intensive therapy program is conducted for a week during the employee ' s normal work hours. D. An injury-specifi c rehabilitation program is provided 3 or 4 days a week for 3 weeks

B: Both abrasions and turf toe (hyperextension of fi rst metatarsophalangeal joint) seem to occur more frequently in athletes using synthetic surfaces. To minimize the risk and severity of skin abrasions, athletes practicing and competing on synthetic turf should wear padding on elbows and knees and cover exposed skin surfaces

Which of the following actions would be most helpful in minimizing injuries common to participation on artifi cial turf? A. Encourage athletes to wear longer cleats. B. Apply extra padding and taping to athletes' exposed skin. C. Encourage athletes to wear heel cups inside their cleats. D. Tape longitudinal arches of each athlete. E. Encourage athletes to wear two pairs of socks with the outside pair inside out.

C: Well-acclimatized athletes sweat earlier in an exercise session compared with nonacclimatized athletes. Although sweat rates vary, well-acclimatized athletes tend to sweat more than nonacclimatized athletes.

Which of the following athletes is demonstrating that he or she is acclimatized to a hot and humid environment? A. The athlete who produces the least amount of sweat after 1 hour of participation B. The athlete who consumes the least amount of water during the practice session C. The athlete who is the fi rst team member to begin sweating during an outdoor practice session D. The athlete who has the highest heart rate during the exercise session E. The athlete who demonstrates the highest percentage of sodium loss on the team

C: Injury can be a signifi cant stressor for athletes, especially if it is season ending, career ending, or severe enough that it would trigger fears of returning to activity

Which of the following athletes would be at risk for a potential psychological concern? A. A freshman who sustains a meniscal tear and will miss 3 to 4 weeks of competition B. A sophomore who is choosing not to join his friends in weekend alcohol use C. A senior who is draft eligible and sustains a femur fracture with 2 weeks left in the season D. A junior who sustained a second concussion that resolved quickly and who has returned to full school and sport activities

B: Custom orthotics are most effective in patients with symptomatic conditions associated with a biomechanical deformity

Which of the following athletes would most benefi t from using custom foot orthotics during sports participation? A. A fi eld hockey player with weak anterior tibialis muscle B. A softball player with pes planus and posterior medial tibial stress syndrome C. A runner with asymptomatic Sever ' s disease D. A soccer player with weak evertors E. A gymnast with pes cavus and a plantar-fl exed fi rst ray

E: An estimated 8,700 health-care workers contract hepatitis B virus (HBV) each year. However, healthcare workers who have received the hepatitis B vaccine and subsequently developed immunity to the virus are at very little risk for developing the infection when exposed.

Which of the following bloodborne pathogens is most commonly contracted by health-care workers? A. Hepatitis C virus (HCV) B. Human papillomavirus (HPV) C. Human immunodefi ciency virus (HIV) D. Hepatitis D virus (HDV) E. Hepatitis B virus (HBV)

D: The National Collegiate Athletic Association (NCAA) bans drugs that fall into these classes: stimulants, anabolic agents, alcohol and beta blockers (rifl e competition only), diuretics and other masking agents, illicit drugs, peptide hormones and analogues, antiestrogens, and beta-2 agonists.

Which of the following classes of drugs are banned by the National Collegiate Athletic Association (NCAA)? A. Cathartics B. Histamine-2 blockers C. Sympathomimetics D. Peptide hormones

A: The air/gel bladder brace design consists of two stirrups made of a thermoplastic outer shell lined with either air or gel bladders to contour to the lower leg and ankle. The two stirrups are connected by an adjustable nylon strap at the plantar surface of the heel

Which of the following commercially available ankle braces are designed to limit only inversion and eversion and are most often used to provide compression and moderate support to an ankle during the acute phase of an ankle injury? A. Air/gel bladder braces B. Wrap braces C. Lace-up braces D. Semirigid braces

D: In athletes 35 years of age and older the leading cause of sudden death is atherosclerotic coronary disease resulting in myocardial infarction. This condition is very uncommon in younger athletes, although it can occur in younger athletes in the presence of family history, hypercholesterolemia, and hypertension.

Which of the following conditions is the leading cause of cardiac problems in middle-aged and older physically active people? A. Cystic fi brosis B. Marfan ' s syndrome C. Cardiac arrhythmia D. Atherosclerotic coronary disease E. Hypertrophic cardiomyopathy

B: Particulate matter in the air, such as pollen, will result in increased airway irritation for persons with asthma, chronic obstructive pulmonary disease, emphysema, and bronchitis

Which of the following conditions would be exacerbated by a high pollen count? A. Hypertension B. Bronchitis C. Diabetes D. Tonsillitis

A: Absence of a paired organ disqualifi es athletes from participation in contact/collision sports.

Which of the following conditions would disqualify a student athlete from participation in football? A. Absence of one kidney B. Mild hypertension C. Inguinal hernia D. Controlled epilepsy E. Sickle cell trait

D: SCAT (Sport Concussion Assessment Tool) is a standardized method for establishing baseline concussion data.

Which of the following could you incorporate into your preparticipation examination to gather baseline concussion data? A. Vital sign assessment B. Upper quarter screening C. Glasgow Coma Scale D. SCAT5 (Sport Concussion Assessment Tool)

C: Cultural sensitivity is being aware of the existence of cultural similarities and differences between people. Cultural sensitivity is important, as it can reduce cultural barriers between health-care professionals and patients

Which of the following demonstrates culturally sensitive communication during a preparticipation examination? A. Reading medical history questions aloud for persons with limited English proficiency B. Understanding that a person ' s limitations in English proficiency strongly correlate with his or her level of intellectual functioning C. Writing all notices and communications in the patient ' s language of origin D. Teaching individuals and families who speak languages other than English key words so that they can better communicate with you

C: The U.S. Centers for Disease Control and Prevention makes recommendations regarding immunizations for children and adults. Vaccines for meningitis, human papillomavirus (HPV), and varicella have been added more recently to the list of available vaccinations.

Which of the following infectious conditions is preventable through vaccination? A. Mononucleosis B. Methicillin-resistant Staphylococcus aureus (MRSA) C. Varicella D. Zika virus (ZIKV)

C: Injury prevalence assesses the number of injuries sustained in a specifi c population.

Which of the following is an examination of injury prevalence data you might gather when analyzing your injury rates? A. Number of injuries sustained during practice sessions B. Number of new ankle injuries sustained by the soccer team during the fall season C. Total number of injuries sustained by the basketball team D. Average number of injuries per athlete per practice session

A: Intrinsic factors are directly related to the athlete and include factors such as age, gender, and physical stature. Extrinsic factors relate to the environment, equipment, type of activity, and amount of exposure to injury.

Which of the following is considered an intrinsic risk factor? A. Being a female high school athlete B. Practicing when the temperature is 85°F and the humidity is 85% C. Running on a cross country wooded trail D. Using an inflatable bladder football helmet E. Participating in a contact sport

B: To properly maintain sports equipment and avoid potential litigation, the manufacturer ' s guidelines for assembling, using, and maintaining sports equipment, including personal protective equipment, should be followed exactly

Which of the following is correct regarding maintenance of sports equipment? A. Repair all defective equipment within a reasonable period of time. B. Strictly follow manufacturer ' s guidelines for equipment maintenance. C. Delegate equipment maintenance to coaches and players using equipment. D. Clean equipment using only Environmental Protection Agency-approved cleaning solutions and disinfectants.

C: Due to decreased activity levels in schools and activities of daily living, a general fi tness and conditioning program is required before the start of competitive sports to decrease risk of injury

Which of the following is recommended for pediatric athletes to minimize overuse injuries? A. Pediatric athletes should have 3 to 4 days off per week. B. Pediatric athletes should participate on a maximum of two teams of the same sport per season. C. Pediatric athletes should begin a general fi tness, conditioning, strengthening, and fl exibility program 2 months before the start of the competitive sports season. D. Pediatric athletes should progress the duration, load, and intensity of workouts by no more than 20% per week.

D: The closed-cell foam would conform to the humerus to allow for force distribution, whereas the thermo-moldable plastic allows for a hard barrier to outside forces.

Which of the following is the best material for constructing a functional protective device to protect a humeral exostosis? A. Plaster cast material and orthopedic felt B. Thermo-moldable plastic and adhesive moleskin C. Fiberglass roll and orthopedic felt D. Thermo-moldable plastic and closed-cell foam E. Adhesive Sorbothane and adhesive felt

B: A solution of bleach and water or a disinfectant approved by the Environmental Protection Agency are recommended cleaning solutions for surfaces

Which of the following is the most effective and cost-effi cient cleaning solution to safely decontaminate soiled surfaces such as treatment tables and fl oors? A. Ammonia:water (1:20) B. Bleach:water (1:10) C. White vinegar:water (1:10) D. Hydrogen peroxide:water (1:20)

E: A disaccharide is a combination of two monosaccharides. Maltose is two glucoses, lactose is a glucose and a galactose, and sucrose is a glucose and a fructose.

Which of the following lists contains only disaccharides? A. Glucose, maltose, lactose B. Lactose, galactose, glucose C. Galactose, fructose, glucose D. Sucrose, fructose, maltose E. Maltose, lactose, sucrose

D: An ergonomic assessment is an evaluation of the employee ' s interaction with a workstation environment. In addition to adjusting workstation settings and making recommendations to the employee, this assessment may include recommended work breaks as well as stretching and strengthening exercises. The most qualifi ed workplace employees should conduct this assessment, make recommendations, and construct the report.

Which of the following people should be included in a worksite ergonomic risk assessment? A. Worker ' s direct supervisor, stockholders in the company, and risk management coordinator B. Worker, director of human resources, and athletic trainer C. Facility nurse, director of human resources, and athletic trainer D. Worker, athletic trainer, and occupational therapist E. Safety engineer, risk management coordinator, and director of facility security

D: This response is consistent with the rules of the game to protect participants

Which of the following persons is/are required to wear a helmet during a baseball game? A. The batter only B. The on-deck batter and the batter only C. The batter, the on-deck batter, and the base coaches only D. The batter, the on-deck batter, the base coaches, and the base runners only E. The batter, the on-deck batter, the base coaches, the base runners, and all the umpires

A: The National Collegiate Athletic Association (NCAA) Women ' s Lacrosse 2018 and 2019 rules manual states: "Section 27. All fi eld players must wear eye protection properly. Eye protection must meet the most current ASTM Specifi cation Standard for eye protectors for women ' s lacrosse and must be listed on the US Lacrosse website

Which of the following pieces of equipment are mandated by the National Collegiate Athletic Association (NCAA)? A. Protective eyewear for female lacrosse players B. Rib protection for baseball catchers C. Protective cups for wrestlers D. Intraoral mouthpiece for male basketball players E. Helmet with face mask for softball batters

C: Extrinsic risk factors are risk factors that occur external to the body and result in injury, such as the blunt force sustained in commotio cordis

Which of the following potential causes of sudden cardiac death is caused by an extrinsic risk factor? A. Hypertrophic cardiomyopathy B. Long QT syndrome C. Commotio cordis D. Myocarditis

C: Lying directly on the heat pack could result in the silicate gel being pushed out between the seams or weak area of the heat pack

Which of the following recommendations regarding moist heat pack use is designed to protect the patient and extend the life of the equipment? A. Heat packs should be removed and washed with soap and water weekly to prevent mold from developing. B. Heat packs should be allowed to cool to room temperature before replacing them in the water bath. C. Heat packs should be placed on a patient rather than allowing the patient to lie directly on the heat pack. D. Heat packs should be used only with a commercial cover as the barrier between the patient and the heat pack.

C: Warm-up, stretching, and cool down are not shown to prevent injury but are commonly accepted methods of minimizing soreness associated with strenuous activity

Which of the following recommendations would be made to an athlete to decrease the risk and intensity of soreness following exercise? A. Warm up and perform static stretching for a minimum of 20 to 30 minutes before exercise. B. Stop every 20 to 30 minutes during the exercise session. C. Complete a short, dynamic warm-up before exercise and an active cool-down afterward. D. Consume 24 oz of an electrolyte beverage before and after activity.

A: Stage 1 and stage 2 hypertension most typically require medication management and are associated with values of systolic measures greater than or equal to 130 mm Hg or diastolic measures greater than or equal to 80 mm Hg

Which of the following serial blood pressure measurements would indicate the need for referral and probably medication management? A. 136/76 mm Hg B. 128/78 mm Hg C. 110/68 mm Hg D. 102/60 mm Hg

C: A functional capacity evaluation is part of a pre- work hardening program assessment. It is used to determine skill defi cits as measured against physical job requirements

Which of the following skills may be evaluated in a functional capacity evaluation completed before developing a work hardening or conditioning program? A. 40 yard sprint time B. Vertical jump height C. Hand grip strength D. 1 minute sit-up maximum

B: When used according to manufacturer ' s guidelines, a football player ' s required protective equipment is not designed to prevent, or cause, a catastrophic spinal cord injury. This injury occurs with structural distortion of the cervical spinal column, which may result from improper tackling methods or poor blocking techniques

Which of the following statements best characterizes prevention of catastrophic cervical spine injuries? A. A football helmet is designed to reduce the risk of axial loading. B. Catastrophic cervical spine injury resulting from axial loading is neither caused nor prevented by players' standard equipment. C. Proper training in tackling and blocking techniques is not as effective as protective equipment in minimizing the risk of a catastrophic cervical spine injury. D. Players who initiate contact with their head down are less at risk for sustaining a catastrophic cervical spine injury if a cowboy collar is added to the shoulder pads. E. Players who are on the receiving end of headdown, helmet-to-helmet contact are not at risk for a catastrophic cervical spine injury.

D: Current research is inconclusive and is limited by a variety of factors, such as small sample size, limited sport exposure, climate, and player position.

Which of the following statements correctly describes the relationship between injury rates and synthetic turf playing surfaces compared with natural grass surfaces? A. Injury rates are higher during the fi rst 1 to 2 years of a new synthetic turf fi eld. B. Injury rates are higher on natural grass surfaces because they are more diffi cult to maintain. C. Injury rates are signifi cantly higher on synthetic turf fi elds due to the higher speeds the players are able to generate. D. Evidence in the literature is inconclusive, and additional research needs to be conducted.

B: An athletic trainer must consider the tape ' s qualities in order to interpret the tape ' s utility. Grade of backing is another way to describe the number of fi bers that make up the tape, whereas quality of the adhesive mass can be tested with moisture. Quality of adhesive mass also dictates the tape ' s ease of removal. The winding tension determines the amount of force required during tape application

Which of the following statements is true regarding selection of tape size and type? A. Elastic adhesive tape is used for bodily areas requiring high tensile strength. B. Tape is qualifi ed based on the grade of backing, the quality of adhesive mass, and the winding tension. C. Large-width tape is used for bodily areas requiring high tensile strengths. D. The more acute the angles required, the wider the tape must be to fi t the contours. E. Elastic tape is used for joints that do not expand with motion.

A: Even though it is a long accepted practice that prophylactic taping is important for injury prevention and treatment, this is not supported by current research.

Which of the following statements most accurately describes the effectiveness of tape in supporting a joint? A. Current research is inconclusive regarding the effectiveness of taping. B. Support is most effective after 15 minutes of warm-up. C. Support is more effective in adolescent athletes compared with senior athletes. D. Taping is a key component in an injury prevention program. E. Effectiveness is attributed solely to its ability to limit range of motion.

A: Ergonomic risk assessments provide information that can be used to change policies or make changes to individual job responsibilities

Which of the following would be an appropriate result of an ergonomic risk assessment? A. A policy is instituted that all workers on the loading dock will be provided with a lumbar stabilization belt and instructions on when and how to use it. B. Customer service specialists are required to sit during their shift to minimize stress on their lower bodies. C. Cameras are installed to monitor workfl ow patterns and identify potential problems. D. Absentee rates are calculated for different departments.

C: Intrinsic risk factors originate within the individual, such as lean-to-fat mass ratio.

Which of the following would be an intrinsic risk factor for exertional heatstroke? A. High ambient temperature and humidity B. Minimal access to fl uids before and during participation C. High muscle mass-to-body fat ratio D. Inappropriate work-to-rest ratios

E: This is the most appropriate selection based on the size of the treatment area and the shoulder spica application procedure

Which of the following wraps should you select to most effectively apply a shoulder spica to a female collegiate volleyball player? A. 6-in. Å~ 10-yd elastic bandage B. 6-in. Å~ 6-yd elastic bandage C. 4-in. adhesive elastic tape D. 6-in. adhesive elastic tape E. 4-in. Å~ 10-yd elastic bandage

A: There are a number of conditioning principles aimed at minimizing risk of exercise-related injury. Progression is related to the controlled and systematic increase in intensity and overload of workouts over time.

Which principle of conditioning is a coach following when he increases the team ' s off-season conditioning program gradually and within each team member ' s abilities to adapt to the overload in order to minimize risk of injury? A. Progression B. Intensity C. Specifi city D. Consistency

B: Cultural sensitivity and professionalism is exhibited when an athletic trainer is willing to listen to an athlete, research the information discussed, and then provide recommendations.

Which response demonstrates an athletic trainer responding with cultural awareness and sensitivity to an athlete who asks that refl exology be incorporated into his sports injury treatment plan? A. "The current sports medicine literature doesn ' t support the effectiveness of refl exology in treating musculoskeletal injuries such as yours." B. "I don ' t know much about refl exology, but tell me why you feel it may be helpful." C. "As a health-care professional I practice evidencebased medicine and avoid grasping at treatment techniques just because they are new or exciting sounding." D. "Since you are from another country you may not understand all aspects of the treatment we are providing you, so feel free to ask questions."

D: Due to playing surface and sport motions, tennis shoes should have a fi rmer sole than running shoes

Which type of activity is best supported by athletic shoes with a rigid sole? A. Running B. Cross training C. Volleyball D. Tennis

C: Tension is generated by creating maximal force against an immovable resistance, so no change in muscle length occurs

Which type of muscular strengthening technique is characterized by an increase in muscle tension without changing the length of the muscle? A. Isotonic B. Isokinetic C. Isometric D. Eccentric

A: Noncantilevered pads allow more shoulder mobility and are recommended for quarterbacks, wide receivers, and youth football players

Which type of shoulder pad is recommended for quarterbacks and wide receivers? A. Noncantilevered B. Cantilevered C. Flatpad D. Streamlined

A: Sorbothane is very dense and is designed to absorb high amounts of energy. It is a specifi c class of foams composed of viscoelastic polymers

Which type of soft orthotic material has a high energy-absorbing quality with a high density, making it effective for preventing blisters and absorbing ground reaction forces in multiple directions? A. Sorbothane B. Orthopedic felt C. Sponge rubber D. Gauze padding E. Closed-cell foam

A: Jet lag symptoms are accentuated when traveling from west to east

Which type of travel is most likely to result in circadian dysrhythmia? A. Travel from California to Massachusetts B. Travel from Louisiana to Colorado C. Travel from Wisconsin to Florida D. Travel from Indiana to Oregon

D: The signs and symptoms of hypoglycemia may be present when blood glucose concentration drops below 60 mg/dL. Hypoglycemia may be due to inadequate glucose (caloric) intake in relation to activity and caloric expenditure

While exercising, a person with type 1 diabetes mellitus begins to experience symptoms of hypoglycemia. Using a glucometer, he assesses his blood glucose level. What reading indicates hypoglycemia? A. 125 mg/dL B. 100 mg/dL C. 75 mg/dL D. 55 mg/dL

B: Because the body has a limited capacity to use amino acids, research suggests a daily protein intake of more than 2 g/kg body weight, as more is unlikely to produce additional muscle gains

While providing nutritional counseling to athletes, you recommend that in addition to consuming protein in a daily balanced diet, they consume protein during and immediately after an exercise session. What is the maximum total daily protein intake that is recommended, based on research suggesting more is unlikely to result in further muscle gains? A. 1.0 g/kg/day B. 2.0 g/kg/day C. 3.0 g/kg/day D. 4.0 g/kg/day

C: Education on proper inhaler use is an important component of an asthma care plan. Improper inhaler use is common and can result in patients receiving incorrect and ineffective dosages.

While reviewing a preparticipation examination medical history, you note an athlete has indicated a previous diagnosis of exercise-induced bronchospasm. He has been using an inhaled beta-2 agonist from a metered-dose inhaler taken 15 to 30 minutes before the onset of each exercise and sport activity session. After speaking with this athlete, you think he needs education on properly using his inhaler. Which of the following should be included in the instructions you provide? A. Shake inhaler for at least 45 seconds before use. B. Exhale through nose for 1 to 3 seconds before placing mouthpiece of inhaler in mouth. C. While pressing down on the metal cartridge, breathe in slowly as far as you can and then hold your breath as long as possible. D. Avoid drinking water or eating food for at least 15 minutes after inhalations.

B: Use and time break down the ethyl vinyl acetate (EVA) used to construct the sole and midsole resulting in decreased shock absorption

Why is it recommended that running shoes be replaced every 350 to 500 miles? A. They are no longer clean enough to meet minimum hygiene standards. B. They lose their shock-absorbing properties. C. The material covering the toe box is likely to be damaged. D. The sole is typically worn through.

D: Scar tissue loses its elastic components and so, when stretched, it may not allow full motion. Excess superfi cial scarring may also be indicative of subcutaneous adhesions

Why might an athlete with a large hypertrophic scar across the anterior aspect of his knee joint secondary to a previous knee surgery have limited fl exion range of motion? A. Excess calcium may be deposited in the joint and the area around the scar. B. The scar may act as a wedge in the joint limiting motion. C. The athlete may have changes to the neurologic structures resulting in infl ammation and pain with movement. D. Scar tissue is more inelastic than skin and may prevent full range of motion.

B: the mandatory equipment for all athletes participating in girls' or women ' s lacrosse is (1) eye guard, American Society for Testing and Materials (ASTM) Lacrosse Standards (National Collegiate Athletic Association [NCAA] and National Federation of High School Sports [NFHS]), and (2) mouth guard (NCAA and NFHS), intraoral design covering all upper teeth (NCAA and NFHS), of any highly visible color (NFHS).

You are an athletic trainer for a medical clinic that provides athletic training community outreach. One of your responsibilities is to assist the director of the city recreation department by providing educational information to sports program participants and parents. You must write an informational letter regarding protective equipment for participants in an introductory girls' lacrosse program. What equipment is mandatory for all athletes participating in high school girls' lacrosse that can be required in a city recreation program? A. Gloves B. Eye guard C. Clear mouth guard D. Elbow pads

C: Ground fault circuit interruptors (GFCIs) are designed to shut off power to an outlet immediately when the electrical current is not fl owing normally to decrease the risk of shock

You are attempting to use your ultrasound machine, but it seems that there is no power to the device. You check the outlet and note that the ground fault current interrupter (GFCI) has been tripped. This is the fi rst time this has occurred. What action should you take? A. Call in an electrician to evaluate the outlet. B. Connect an extension cord from the outlet to the ultrasound machine to distribute the voltage. C. Check to see that there are not multiple devices plugged into the outlet, press the reset switch, and continue with your treatment. D. Unplug all devices until the facility can be inspected by your fi re safety offi ce.

B: It is important that coaches and athletic trainers demonstrate mutual respect for one another. Early notifi cations of possible changes to a practice allow the coach to better prepare and ensure that athlete safety takes priority.

You are concerned that severe thunderstorms may pass through your area during the afternoon practice sessions. What actions should you take to maximize athlete safety and minimize communication problems with the coaching staff? A. Notify all the coaches as soon as inclement weather is sighted. B. Discuss with the coaches early in the day that weather may impact practice, review procedures, and provide updates as the weather system approaches. C. Send the coaches an e-mail that weather may impact practices later in the day. D. Speak to each of the coaches as they are beginning their practice and let them know you will be monitoring the weather and will tell them if they need to leave the fi eld.

D: Defi nitions of hypohydration and strategies for avoiding hyponatremia and dehydration should be provided so that participants can prepare

You are coordinating medical care for a large road race that includes 5K, 10K, and half-marathon events. What key educational information should be included in participant guidelines regarding fl uid replacement? A. Participants should be encouraged to drink as much as possible in the 48 hours leading up to the race. B. Participants should be encouraged to drink fl uids at every water station on the course. C. Participants should be provided with a copy of the event safety plan and locations of every water station. D. Participants should be provided with strategies to avoid overconsumption of water and dehydration

D: The efficacy of taping versus bracing is inconclusive, but bracing may provide more consistent support because tape loosens over time. Bracing long-term is more cost-effective in the long term, and the athlete can apply it himself

You are counseling an athlete with a history of multiple ankle sprains. The athlete's father is insistent that his son be taped every day before activity, but you think that the athlete would benefit from a commercial lace-up brace. What rationale would you provide for your recommendation? A. Bracing has been shown to increase performance compared with taping. B. Taping takes too much time and uses too many supplies. C. Research has shown that taping provides signifi cantly less protection against injury than bracing. D. Bracing will provide more consistent support over the course of the practice and over time is more cost and time effi cient.

A: If the athlete can fully raise the arms overhead without the pads sliding, the neck opening is the correct size

You are educating your new volunteer equipment manager about appropriate football shoulder pad fi tting. Which of the following statements indicates the shoulder pads are fi tted correctly? A. The athlete can raise his arms overhead without the pads sliding. B. The epaulets cover the acromioclavicular joint. C. The underarm straps are snug enough that it is diffi cult to slide a fi nger underneath. D. The pads shift slightly when the jersey is put on, but the jersey holds them in position.

A: An equipment manufacturer is liable for any injury resulting from defective equipment or equipment inadequate for its intended purpose. However, when any piece of protective sports equipment is modifi ed in any way, the manufacturer ' s liability is voided, and the individual who modifi ed the equipment becomes liable for any damages and injuries sustained

You are preparing your college football team for the upcoming season. A football player is having diffi culty fi nding a pair of shoulder pads that fi t him correctly. As his athletic trainer, you decide to modify a pair of shoulder pads to fi t this athlete. Which modifi cations are least likely to result in you or your institution being involved in a lawsuit should the athlete become injured? A. Limit your modifi cations to only those recommended by the shoulder pad manufacturer. B. Limit your modifi cations to only those that you have made in the past that did not result in any injuries. C. Limit your modifi cations to those that your athletic director agrees in writing to take responsibility for should a player get injured. D. Limit your modifi cations to only those you have seen colleagues make that did not result in any injuries. E. Limit your modifi cations to only those specifi c to the player ' s position.

A: This method of examination scheduling would allow for the highest quality and most individualized up-to-date information for each athlete.

You are the athletic trainer at a high school that sponsors 22 sports. There are approximately 500 athletes. For this high school, which preparticipation examination format is the most effective and effi cient? A. Provide a station-based examination with the assistance of multiple medical and health-care professionals before the start of each sport season (i.e., fall, winter, spring). B. Provide individual examinations with the assistance of fi ve family practice physicians stationed in individual examination rooms. C. Provide a station-based examination with the assistance of multiple medical and health-care professionals in the summer before the start of the school year. D. During the summer months, schedule individual appointments for each athlete with the team physician. E. Require each athlete to obtain on his or her own a physical examination through a licensed health-care provider.

C: Snellen eye charts assess distance visual acuity, which is normally 20/20. The numerator represents the distance the patient is standing from chart (20 feet), and the denominator represents the distance at which a normal person can read the same-size letters on that line. The higher the denominator, the poorer the patient ' s distance vision. Athletes with suboptimal visual acuity should be referred for evaluation and possible correction

You are using a Snellen eye chart to assess baseline visual acuity. An athlete is able to comfortably read the line associated with 20/40 vision in the right eye and 20/50 vision in the left eye. What action would you take? A. No action is necessary. B. Retest the athlete serially for 2 to 3 weeks to see if you get the same results. C. Refer the athlete to an optometrist for evaluation. D. Hold the athlete from all participation pending referral results.

C: The National Athletic Trainers' Association (NATA) offi cial statement on automated external defi brillators (AEDs) states that an AED program should be part of an athletic trainer ' s emergency action plan and an AED should be standard emergency equipment.

You believe your school ' s athletic department should purchase an automated external defi brillator (AED). What document should be used to encourage your school to include an AED as part of your emergency equipment? A. National Athletic Trainers' Association (NATA) Heat Stroke Treatment Authorization Form B. NATA Position Statement: Management of Sport Concussion C. NATA Offi cial Statement: Automated External Defi brillators D. NATA Heads up video

D: Dimension-specifi c instruments assess one specifi c aspect of a patient ' s health and relate closely to psychological well-being

You elect to have a patient complete a McGill Pain Questionnaire before his or her rehabilitation appointment. Which type of patient-reported outcome (PRO) scale is being used? A. Generic instrument B. Disease-specific instrument C. Site-specific instrument D. Dimension-specific instrument

B: Standardized Assessment of Concussion (SAC) measures cognitive function by assessing orientation, immediate memory, concentration, and delayed recall.

You elect to utilize the SCAT5 (Sport Concussion Assessment Tool) to establish baseline data. Which portion of the SCAT5 assesses cognitive function? A. Glasgow Coma Scale (GCS) B. Standardized Assessment of Concussion (SAC) C. Maddocks score D. Balance Error Scoring System (BESS)

D: The types of errors recorded during the balance examination component of the SCAT5 (Sport Concussion Assessment Tool) are based on the modifi ed Balance Error Scoring System (BESS). The modifi ed BESS score is calculated by adding 1 error point for each error during the three 20-second tests

You elect to utilize the SCAT5 (Sport Concussion Assessment Tool) to establish baseline data. Which type of error should be counted when conducting the double leg stance component of the balance examination? A. Hands lifted off of shoulders B. Flexing knees greater than 20° C. Moving hips into greater than 5° abduction D. Remaining out of test position greater than 5 seconds

C: The Dietary Guidelines and Physical Activity Guidelines provide clear guidelines and strategies specific to the general population. These are required by law to be updated regularly

You have been asked to speak at a local Rotary Club meeting regarding recommendations for improving nutrition and physical activity for the general population. Which of the following resources might you share with your audience? A. National Strength and Conditioning Association website, which is updated regularly B. Healthy People 2020 objectives, which are updated every 10 years C. Dietary Guidelines for Americans and Physical Activity Guidelines for Americans, which are updated regularly D. American College of Sports Medicine position statements, which are updated periodically as information changes

B: It is the responsibility of the athletic trainer to make sure that an athlete is using protective equipment in the right way and that the athlete is legally protected in the event that the equipment fails

You have been informed that one of your lacrosse players has modifi ed his shoulder pads to make them more comfortable. What action should you take? A. No action is necessary as long as the athlete is comfortable. B. Meet with the athlete and explain that modifying equipment in any way may limit its protective ability and will invalidate the manufacturer ' s warranty. C. Notify the coach that the player should be withheld from activity until the equipment modifi cation is addressed. D. Inspect the equipment in question and as long as the athlete is protected no additional action is necessary.

D: Elastic wraps are used to effectively support soft tissue or joints and to hold dressings or pads in place

You have created a hard shell pad to protect an athlete returning to play following a hip pointer injury. How might this pad best be held in place? A. Secured directly to the skin with nonelastic adhesive tape B. Secured under compression shorts C. Secured with prewrap and elastic tape wrapped around both hips D. Secured with a hip spica using an elastic bandage and tape

A: An aching sensation is an indicator of overheating. While sweating may result in a burn, that is a normal sensation for this treatment, as is warmth. Motor fi bers are not stimulated to contract the muscle.

You have selected shortwave diathermy to increase tissue temperature in a patient ' s hamstring. Before treatment, you communicate to the patient expected sensations. Which of the following statements made by the patient during the treatment might indicate that the tissue is being overheated? A. "I feel a really deep ache in my hamstring." B. "I feel like the muscle is contracting." C. "It feels really, really hot, but not burning." D. "I feel like I am starting to sweat."

A: According to the National Athletic Trainers' Association (NATA) position statement, trauma, even in persons without diabetes, often causes a hyperglycemic state, which can impair wound healing. Blood glucose should be frequently monitored following trauma.

You receive a call from your team physician alerting you that the female equestrian athlete you had referred to him has been diagnosed with type 1 diabetes mellitus. You decide it is important to counsel the athlete on the impact of the injury on her disease. Which statement is an accurate representation of the effect that injury may have on glycemic control? A. Athletes with type 1 diabetes appear to have an exaggerated glycemic response to trauma. B. Injury does not affect an athlete with type 1 diabetes any differently than an athlete without type 1 diabetes. C. Athletes with type 1 diabetes are at an increased risk for infection. D. Athletes with type 1 diabetes, even those with poorly controlled blood glucose, will demonstrate fracture healing at a similar rate to athletes without diabetes.

E: Overtraining, or training too hard for too long without proper rest, can lead to staleness and eventual burnout. Staleness can manifest itself in both physiological and psychological symptoms, including elevated resting heart rate and blood pressure, sleeping disorders, loss of appetite, perceived body pains, and mood disorders

You suspect that a swimmer may be experiencing symptoms of overtraining. Which of the following situations is a signifi cant indicator of overtraining? A. The athlete demonstrates increased whole-body muscle tone. B. The athlete complains of being easily agitated. C. The athlete reports increased urinary output. D. The athlete ' s blood pressure is below preparticipation baseline. E. The athlete ' s resting heart rate is elevated above preparticipation baseline.

A: Polycarbonate lenses are the best material for corrective lenses because the polycarbonate has great strength, and the lenses are almost unbreakable.

Your athlete is unable to wear contact lenses. Corrective eyeglass lenses made of which of the following materials would you recommend for participation? A. Polycarbonate lenses B. Plastic lenses C. Polyethylene lenses D. Fiberglass lenses E. Ethyl vinyl acetate lenses

B: Each of these answers are a recommended component of a comprehensive proactive emergency action plan (EAP) specifi c to lightning safety. Reliable means of monitoring the weather should be identifi ed and used, a person should be appointed to have the authority to suspend activity, and safe locations should be identifi ed ahead of time. However, the component that is most useful in deciding if play will be suspended is having in place specifi c criteria for suspending and resuming play.

Your baseball team is playing a doubleheader and you are only one inning into the second game. Dark clouds are moving into the area, and you have heard some rumbles of thunder in the distance. The baseball team ' s coach also notes the weather changes and asks you if play will be suspended. Which component of your venue-specifi c comprehensive proactive emergency action plan (EAP) specifi c to lightning safety is the most helpful for answering the coach ' s question? A. The plan identifi es safe locations from the lightning hazard. B. The plan identifi es specifi c criteria for suspending and resuming activity. C. The plan identifi es a specifi c person (by name or job title) who makes the decision to suspend and resume sport participation D. The plan identifi es a reliable means of monitoring the weather.

D: A Valsalva maneuver results from performing an isometric contraction while holding the breath, as occurs with "bearing down." A Valsalva maneuver causes an increase in blood pressure

Your client is using isometric strengthening exercises as part of a work hardening program and has been instructed to breathe while performing the exercises. Why is breathing during isometric exercises recommended? A. To maximize blood fl ow to the client ' s working muscles B. To control the client ' s heart rate C. To minimize accumulation of metabolic by-products in the client ' s working muscles D. To avoid a rapid increase in the client ' s blood pressure

C: Neoprene sleeve braces should be washed in cold water with mild detergent, rinsed, and then air dried

Your patient has been provided a neoprene sleeve brace for support for her patellofemoral joint condition. Which of the following is a manufacturer brace care recommendation for this type of brace? A. Wash in hot water using a mild detergent. B. Dry using a hot air laundry dryer. C. Air dry. D. Wash in hot water using a 10:1 bleach solution.

B: Trauma to any exposed lower leg bone or soft tissue can result in signifi cant damage and pain. The regular use of properly fi tted shin guards (from malleoli to just below tibial tuberosity) in all practice and games should be encouraged

Your school ' s annual injury summary indicates your men ' s soccer team sustained a higher number of anterior lower leg injuries this past year compared with the previous fi ve seasons. An e-mail to the coaching staff explaining the importance of appropriate shin guard sizing and usage is indicated. What should be included in this communication? A. For maximum protection, shin guards should cover the anterior lower leg from the tibial tuberosity to midtibia. B. For maximum protection, shin guards should cover the anterior lower leg from just above the malleoli of the ankle to just below the tibial tuberosity. C. For maximum protection, shin guards should cover the anterior and lateral lower leg from the distal third of the tibia to the superior tip of the gastrocnemius muscle. D. For maximum protection, the largest shin guard size available should be worn by all players.

A: The National Weather Service sets standard defi nitions for weather conditions and is considered a viable resource

Your softball team is preparing to practice when you notice the sky darkening. You remember that the weather forecast mentioned possible thunderstorms for today. You consult the National Weather Service and discover that a severe thunderstorm watch has been declared for your area. What does this declaration mean? A. Conditions are favorable for the development of severe weather. B. Severe weather has been detected in your area. C. Steps should be taken to remove athletes from the fi eld. D. All persons should take necessary precautions to preserve their own safety

C: Assuming local time on arrival promotes easier adaptation to minimize negative impacts of jet lag.

Your tennis team will be traveling from the east coast to the west coast for a tournament. What guidance might you give the coach to minimize the negative impacts of jet lag on the players? A. Do not allow the players to drink caffeine while traveling west. B. Ask the players to stay up late before you leave so they will sleep on the plane. C. On arrival, immediately assume the local time schedule for eating, practicing, and sleeping. D. Have the players keep their watches set on the time at home to improve the ability to adjust on return.

B: This is in compliance with the rules for mandatory equipment as outlined by the National Collegiate Athletic Association (NCAA) Rules Committee

Your volleyball setter is returning to participation following a distal radius fracture. The team physician has requested that she wear a hard splint for the fi rst 2 weeks. What steps must be taken so that she can play? A. The head referee must approve the hard splint before the match. B. The splint must be covered on every surface in 1⁄2-in.-thick slow rebounding foam. C. The splint can be worn in practice but may not be worn in matches. D. The athlete will be unable to participate until the splint is removed.


Ensembles d'études connexes

INTRO TO LIFETIME FITNESS EXAM STUDYGUIDE

View Set

Systems of Linear Equations Elimination Method

View Set

AMERICAN NATIONAL GOVERNMENT TEST 2 STUDY GUIDE

View Set

Foundations for Living - The Infallibility of Scripture

View Set

Series 66: Session 12 - Method of Quantitative Analysis

View Set

Independent and Dependent variables: Examples

View Set

Personal Finance, Chapter 3, Terms

View Set

Risk and loss, quiz 1, and overview

View Set